Извлечение квадратного корня из обеих частей уравнения

Если из обеих частей уравнения извлечь квадратный корень, то полýчится уравнение равносильное исходному.

Рассмотрим следующее уравнение:

x2 = 16

Это простейшее квадратное уравнение, имеющее два корня: 4 и −4. Такое уравнение мы решали используя определение квадратного корня.

Согласно определению квадратного корня, число b является квадратным корнем из числа a, если ba и обозначается как = √a.

Тогда в случае x2 = 16, можно записать что = √16, откуда = ±4.

Теперь решим данное квадратное уравнение путем извлечения квадратного корня из обеих частей уравнения.

«Обернём» обе части уравнения x2 = 16 в квадратный корень:

извление квадратного корня из обеих частей уравнения рис 1

Теперь вспоминаем одно из свойств квадратного корня, которое гласит что квадратный корень из квадрата числа равен модулю этого числа

кор из а в 2 равно а

Тогда в левой части нашего уравнения получим модуль из x, а в правой части число 4

извление квадратного корня из обеих частей уравнения рис 2

Получили простейшее уравнение с модулем. Оно имеет два корня: 4 и −4. Запишем это решение в виде совокупности уравнений:

извление квадратного корня из обеих частей уравнения рис 4

Проверка:

извление квадратного корня из обеих частей уравнения рис 5

Из правой части уравнения x2 = 16 следует извлекать именно арифметический квадратный корень. Ранее мы говорили, что квадратный корень имеет два значения: положительное и отрицательное. То есть:

извление квадратного корня из обеих частей уравнения рис 9

Но в данном случае нас интересует именно неотрицательное значение 4 (его и называют арифметическим квадратным корнем). Потому что если мы извлечем и второй корень (отрицательный −4), то получим уравнение |x|= −4 которое не имеет решений.


Пример 2. Решить уравнение 3x= 12

Решение

Разделим обе части на 3

извление квадратного корня из обеих частей уравнения рис 6

Извлечём квадратный корень из обеих частей получившегося уравнения:

извление квадратного корня из обеих частей уравнения рис 7

Получили простейшее уравнение с модулем. Решим его, сведя его в совокупность:

извление квадратного корня из обеих частей уравнения рис 8

Ответ: 2 и −2.


Пример 3. Решить уравнение (+ 2)= 25

Решение

Извлечём квадратный корень из обеих частей получившегося уравнения:

извление квадратного корня из обеих частей уравнения рис 9

Решим получившееся уравнение с модулем:

извление квадратного корня из обеих частей уравнения рис 10

Ответ: 3 и −7.


Пример 4. Решить уравнение x2 − 10 = 39

Решение

Перенесем −10 в правую часть изменив знак:

извление квадратного корня из обеих частей уравнения рис 11

Извлечём квадратный корень из обеих частей получившегося уравнения:

извление квадратного корня из обеих частей уравнения рис 12

Решим получившееся уравнение с модулем:

извление квадратного корня из обеих частей уравнения рис 13

Ответ: 7 и −7.


Задания для самостоятельного решения

Задание 1. Решить уравнение:
Решение:
Ответ: 9 и −9.
Задание 2. Решить уравнение:
Решение:
Ответ: 0,4 и −0,4.
Задание 3. Решить уравнение:
Решение:
Ответ: 4 и −4.
Задание 4. Решить уравнение:
Решение:
Ответ: и .
Задание 5. Решить уравнение:
Решение:
Ответ: и .

Понравился урок?
Вступай в нашу новую группу Вконтакте и начни получать уведомления о новых уроках

Возникло желание поддержать проект?
Используй кнопку ниже

Решение неравенств с модулем методом интервалов

Если неравенство содержит два и более модуля, его удобнее решать методом интервалов.

Процесс решения неравенств с модулем методом интервалов во многом похож на процесс решения уравнений с модулем методом интервалов.

Рассмотрим несколько примеров.

Пример 1. Решить неравенство |7 − x|+|2+ 3|< 16

Решение

Для начала находим такие x, при которых подмодульные выражения 7 − x и 2+ 3 обращаются в ноль. Для этого приравняем эти выражения к нулю и решим простейшие линейные уравнения:

неравенства с модулем метод интервалов рис 1

Отметим числа 7 и  минус три вторых на координатной прямой. Мéньшие числа отмечаем левее, бóльшие правее:

неравенства с модулем метод интервалов рис 2

Получили три промежутка: неравенства с модулем метод интервалов рис 4  ,  неравенства с модулем метод интервалов рис 5 и неравенства с модулем метод интервалов рис 6 Теперь необходимо решить исходное неравенство на каждом из этих промежутков. Надо иметь ввиду, что на каждом из этих промежутков модули исходного неравенства могут раскрываться по-разному.

Решим исходное неравенство на первом промежутке неравенства с модулем метод интервалов рис 4

Далее рассуждаем так:

Если неравенства с модулем метод интервалов рис 4, то при любом значении на данном промежутке подмодульное выражение 7 − x станет неотрицательным, а значит модуль |7 − x| на промежутке неравенства с модулем метод интервалов рис 4 будет раскрываться со знаком плюс. Второй модуль |2+ 3| на промежутке неравенства с модулем метод интервалов рис 4 будет раскрываться со знаком минус.

Тогда в результате раскрытия модулей на промежутке неравенства с модулем метод интервалов рис 4 исходное неравенство примет вид:

неравенства с модулем метод интервалов рис 9

Решим данное неравенство:

неравенства с модулем метод интервалов рис 7

Итак, сейчас мы рассматриваем промежуток неравенства с модулем метод интервалов рис 4. И решив на этом промежутке исходное неравенство мы получили неравенство > −4.

Теперь начинается самое интересное. Надо выяснить выполняется ли неравенство > −4 на промежутке неравенства с модулем метод интервалов рис 4. Или задать такой вопрос: «при каких значениях промежутка неравенства с модулем метод интервалов рис 4 выполняется неравенство > −4»

Для наглядности нарисуем еще одну координатную прямую и изобразим на ней решения неравенства > −4 и неравенства с модулем метод интервалов рис 4

неравенства с модулем метод интервалов рис 8

На рисунке видно при каких значениях промежутка неравенства с модулем метод интервалов рис 4 выполняется неравенство > −4. Эти значения лежат в промежутке от −4 до минус три вторых

Значит первым нашим решением будет промежуток от −4 до минус три вторых

неравенства с модулем метод интервалов рис 20

Решим теперь исходное неравенство на промежутке неравенства с модулем метод интервалов рис 5

Если неравенства с модулем метод интервалов рис 5, то при любом значении на данном промежутке подмодульное выражение 7 − x станет неотрицательным, а значит модуль |7 − x| на промежутке неравенства с модулем метод интервалов рис 5 будет раскрываться со знаком плюс. Второй модуль |2+ 3| на промежутке неравенства с модулем метод интервалов рис 5 тоже будет раскрываться со знаком плюс.

После раскрытия модулей на промежутке неравенства с модулем метод интервалов рис 5 исходное неравенство примет вид:

неравенства с модулем метод интервалов рис 10

Решим данное неравенство:

неравенства с модулем метод интервалов рис 11

Cейчас мы рассматриваем промежуток неравенства с модулем метод интервалов рис 5. И решив на этом промежутке исходное неравенство мы получили неравенство x < 6. Теперь надо выяснить выполняется ли неравенство x < 6 при неравенства с модулем метод интервалов рис 5

неравенства с модулем метод интервалов рис 13

Неравенство x < 6 выполняется не на всём промежутке неравенства с модулем метод интервалов рис 5, а лишь на промежутке минус три вторых до 6. Запишем наше второе решение:

неравенства с модулем метод интервалов рис 21

Решим теперь исходное неравенство на последнем промежутке неравенства с модулем метод интервалов рис 6

Если неравенства с модулем метод интервалов рис 6, то при любом значении на данном промежутке подмодульное выражение 7 − x станет отрицательным, а значит модуль |7 − x| на промежутке неравенства с модулем метод интервалов рис 6 будет раскрываться со знаком минус. Второй модуль |2+ 3| на промежутке неравенства с модулем метод интервалов рис 6 будет раскрываться со знаком плюс.

Тогда в результате раскрытия модулей на промежутке неравенства с модулем метод интервалов рис 6 исходное неравенство примет вид:

неравенства с модулем метод интервалов рис 14

Решим данное неравенство:

неравенства с модулем метод интервалов рис 15

Cейчас мы рассматриваем промежуток неравенства с модулем метод интервалов рис 6. И решив на этом промежутке исходное неравенство мы получили неравенство неравенства с модулем метод интервалов рис 17, Теперь надо выяснить выполняется ли неравенство неравенства с модулем метод интервалов рис 17 при неравенства с модулем метод интервалов рис 6

неравенства с модулем метод интервалов рис 16

Мы видим, что неравенство неравенства с модулем метод интервалов рис 17 не выполняется ни при каких значениях промежутка неравенства с модулем метод интервалов рис 6. Это значит, что исходное неравенство на промежутке неравенства с модулем метод интервалов рис 6 решений не имеет.

Действительно, возьмём любое число из промежутка неравенства с модулем метод интервалов рис 6, например, число 9 и подставим его в исходное неравенство. В результате получим неравенство которое не выполняется:

неравенства с модулем метод интервалов рис 18

Теперь нужно собрать воедино ответы, которые мы получили на каждом промежутке. Чтобы сделать это, просто объединим промежутки неравенства с модулем метод интервалов рис 22 и неравенства с модулем метод интервалов рис 23

неравенства с модулем метод интервалов рис 24

Ответ: (−4 ; 6).


Пример 2. Решить неравенство: 3|− 2|+|5− 4| ≤ 10

Решение

Найдём x, при которых подмодульные выражения − 2 и 5− 4 обращаются в ноль. Для этого приравняем эти выражения к нулю и решим простейшие линейные уравнения:

неравенства с модулем метод интервалов рис 25

Отметим числа 2 и неравенства с модулем метод интервалов рис 26 на координатной прямой:

неравенства с модулем метод интервалов рис 27

Решим исходное неравенство на промежутке неравенства с модулем метод интервалов рис 28. Оба модуля на данном промежутке раскрываются с минусом:

неравенства с модулем метод интервалов рис 29

Полученное неравенство ≥ 0 выполняется не на всем промежутке неравенства с модулем метод интервалов рис 28, а только на промежутке от 0 до неравенства с модулем метод интервалов рис 26

неравенства с модулем метод интервалов рис 38

Решим теперь исходное неравенство на следующем промежутке неравенства с модулем метод интервалов рис 30. На данном промежутке модуль|− 2| раскрываются с минусом, а модуль |5− 4| с плюсом:

неравенства с модулем метод интервалов рис 31

Полученное неравенство x ≤ 4 выполняется на всём промежутке неравенства с модулем метод интервалов рис 30. Значит на промежутке неравенства с модулем метод интервалов рис 30 исходное неравенство имеет следующее решение:

неравенства с модулем метод интервалов рис 39

Решим исходное неравенство на следующем промежутке ≥ 2. Оба модуля на данном промежутке раскрываются с плюсом:

неравенства с модулем метод интервалов рис 32

Полученное неравенство неравенства с модулем метод интервалов рис 33 выполняется не на всем промежутке ≥ 2, а только на промежутке от 2 до пять вторых

неравенства с модулем метод интервалов рис 40

Запишем окончательный ответ. Для этого объединим промежутки неравенства с модулем метод интервалов рис 34, неравенства с модулем метод интервалов рис 41 и неравенства с модулем метод интервалов рис 35 неравенства с модулем метод интервалов рис 36

Ответ: неравенства с модулем метод интервалов рис 37.


Задания для самостоятельного решения

Задание 1. Решить неравенство:
Решение:
Ответ: .
Задание 2. Решить неравенство:
Решение:
Ответ: решений нет.
Задание 3. Решить неравенство:
Решение:
Ответ: .
Задание 4. Решить неравенство:
Решение:
Ответ: .
Задание 5. Решить неравенство:
Решение:
Ответ: .

Понравился урок?
Вступай в нашу новую группу Вконтакте и начни получать уведомления о новых уроках

Возникло желание поддержать проект?
Используй кнопку ниже

Неравенства с модулем

Продолжаем изучать модуль числа. Сегодня мы научимся решать неравенства с модулем.

Чтобы решать неравенства с модулем, нужно прежде всего уметь решать простейшие линейные неравенства, а также знать что такое модуль и как его раскрывать.

Независимо от того, решаем мы уравнение или неравенство, нужно уметь раскрывать модуль.

Рассмотрим к примеру простейшее неравенство с модулем:

|x| > 2

Чтобы решить данное неравенство раскроем его модуль.

Если подмодульное выражение больше или равно нулю, то исходное неравенство примет вид:

> 2

Решением этого неравенства является множество всех чисел, бóльших 2. Отметим их на координатной прямой:

неравенство с модулем рис 1

А если подмодульное выражение меньше нуля, то исходное неравенство примет вид:

x > 2

Умнóжим обе части этого неравенства на −1. Тогда полýчим неравенство < −2. Решением этого неравенства является множество всех чисел, мéньших −2. Отметим эти решения на том же рисунке, где мы отметили решения для неравенства > 2

неравенство с модулем рис 2

Забавно, но получившиеся промежутки < −2 и > 2 являются ответом к нашей задаче. Если в исходное неравенство |x| > 2 подставить какое-нибудь значение x, удовлетворяющее данному неравенству, то это значение будет принадлежать промежутку (−∞ ; −2) или промежутку (2 ; +∞).

То есть решением исходного неравенства является совокупность из < −2 и > 2

неравенство с модулем рис 12

Совокупностью неравенств мы будем называть несколько неравенств, объединённых квадратной скобкой, и которые имеют множество решений, удовлетворяющих хотя бы одному из неравенств, входящих в данную совокупность.

Чтобы записать окончательный ответ, промежутки < −2 и > 2 следует объединить. В математике знаком объединения служит ∪. Тогда:

x ∈ (−∞ ; −2) ∪ (2 ; +∞)

Знак объединения читается как «или». Тогда запись x ∈ (−∞ ; −2) ∪ (2 ; +∞) можно прочитать так:

Значение переменной x принадлежит промежутку (−∞ ; −2) или промежутку (2 ; +∞)

Действительно, если подставить какое-нибудь значение x, являющееся решением исходного неравенства, то это значение будет принадлежать промежутку (−∞ ; −2) или промежутку (2 ; +∞).

Например, число 3, является решением исходного неравенства |x| > 2

|3| > 2      3 > 2

Значение 3 принадлежит промежутку (2 ; +∞). Также оно удовлетворяет хотя бы одному из неравенств совокупности неравенство с модулем рис 12, а именно неравенству x>2.

Значение −4 тоже является решением исходного неравенства |x| > 2. Это значение принадлежит промежутку (−∞ ; −2)

|−4| > 2        4 > 2

Также значение −4 удовлетворяет хотя бы одному из неравенств совокупности неравенство с модулем рис 12, а именно неравенству < −2.

Согласно определению, модуль числа x есть расстояние от начала координат до точки x. В неравенстве |x| > 2 это расстояние больше чем 2.

Действительно, от начала координат (точка 0) любое расстояние бóльшее двух, будет решением неравенства |x| > 2

неравенство с модулем рис 7

Ответ: x ∈ (−∞ ; −2) ∪ (2 ; +∞)

Обратите внимание, что границы −2 и 2 не включены в соответствующие промежутки. Это потому, что при подстановке этих чисел в исходное неравенство, получается неверное неравенство.

Теперь немного поменяем наш пример. В неравенстве|x| > 2 поменяем знак > на знак <

|x| < 2

Решим это неравенство.

Как и раньше для начала раскрываем модуль. Если подмодульное выражение больше или равно нулю, то получим неравенство < 2. Решениями этого неравенства являются все числа, мéньшие двух. Отметим их:

А если подмодульное выражение меньше нуля, то получим неравенство < 2. Умнóжим обе части этого неравенства на −1. Тогда получим неравенство > −2. Решениями этого неравенства являются все числа, бóльшие −2. Отметим эти решения на том же рисунке, где мы отметили решения для неравенства x < 2.

Для наглядности, решения неравенства > −2 отметим красным цветом:

неравенство с модулем рис 10

Если выражение |x| это расстояние от начала координат до точки x, то неравенство |x| < 2 говорит, что это расстояние меньше чем 2. На рисунке видно, что от начала координат расстояния, мéньшие двух, лежат в промежутках от −2 до 0 и от 0 до 2

неравенство с модулем рис 10

А эти расстояния одновременно будут принадлежать промежуткам < 2 и > −2

неравенство с модулем рис 13

Обратите внимание, что в этот раз промежутки обрамлены знáком системы, а не знáком совокупности как в прошлом примере. Это означает, что значения x одновременно удовлетворяют обоим неравенствам (промежуткам < 2 и > −2)

То есть решением неравенства |x| < 2 является пересечение промежутков < 2 и > −2. Напомним, что пересечением двух промежутков является промежуток, состоящий из чисел, которые принадлежат как первому промежутку так и второму:

x ∈  (−2 ; 0) ∩ (0 ; 2)

Знак пересечения ∩ читается как «и». Тогда запись ∈ (−∞ ; 2) ∩ (−2 ; +∞) можно прочитать так:

Значение переменной x одновременно принадлежит промежутку (−∞ ; 2) и промежутку (−2 ; +∞)

Действительно, если подставить какое-нибудь значение x, являющееся решением неравенства |x| < 2, то это значение будет принадлежать одновременно промежутку (−∞ ; 2) и (−2 ; +∞).

Например, число 1 является решением исходного неравенства |x| < 2

|1| < 2      1 < 2

Значение 1 одновременно принадлежит промежутку  (−∞ ; 2) и промежутку (−2 ; +∞)

неравенство с модулем рис 11

Также, значение 1 удовлетворяет обоим неравенствам системы неравенство с модулем рис 13

А если к примеру подставить значение, не являющееся решением неравенства |x| < 2, то это значение не будет одновременно принадлежать промежуткам (−∞ ; 2) и (−2 ; +∞). Например, значение 7

|7| < 2      7 < 2

Несмотря на то, что значение 7 принадлежит одному из промежутков, а именно промежутку (−2 ; +∞), данное значение не является решением исходного неравенства, поскольку оно не удовлетворяет ему. Также, данное значение не принадлежит одновременно обоим промежуткам: (−∞ ; 2) и (−2 ; +∞).

Для неравенства |x| < 2 ответ можно записать покороче:

x ∈ (−2 ; 2)

Из рассмотренных примеров видно, что решением неравенства с модулем может быть либо объединение промежутков либо их пересечение.

В первом примере мы решили неравенство |x| > 2, то есть неравенство вида |x| > a. Это неравенство при котором модуль больше какого-нибудь числа или буквенного выражения. Решением такого неравенства является объединение решений неравенств, получающихся после раскрытия модуля исходного неравенства. Неравенства, получающиеся после раскрытия модуля, следует записывать в виде совокупности:

неравенство с модулем рис 14

Совокупность свóдится потому, что итоговые решения будут удовлетворять хотя бы одному из неравенств, полученных после раскрытия модуля исходного неравенства.

Во втором примере мы решили неравенство |x| < 2, то есть неравенство вида |x| < a. От предыдущего неравенства оно отличается только знáком. Но это неравенство при котором модуль меньше какого-нибудь числа или буквенного выражения. Решением такого неравенства является пересечение решений неравенств, получающихся после раскрытия модуля исходного неравенства. Неравенства, получающиеся после раскрытия модуля, следует записывать в виде системы:

неравенство с модулем рис 15

Система записывается потому, что итоговые решения будут удовлетворять обоим неравенствам, полученным после раскрытия модуля исходного неравенства.

Эти же правила сохраняются и для неравенств, содержащих знаки ≥ и ≤

Например, решим неравенство |x| ≥ 1. Модуль больше или равен числу. Поэтому решением будет объединение решений неравенств, которые получатся после раскрытия модуля. После раскрытия модуля и выполнения необходимых тождественных преобразований, получим совокупность неравенств  1 и  −1

неравенство с модулем рис 16

неравенство с модулем рис 17

Решением служит объединение промежутков  −1 и  1

x ∈ (−∞ ; −1] ∪ [1 ; +∞)

Обратите внимание, что границы −1 и 1 включены в соответствующие промежутки. Это потому что при подстановке этих чисел в исходное неравенство, получается верное неравенство.

Решим теперь к примеру неравенство |x|  1. Модуль меньше или равен числу. Поэтому решением будет пересечение решений неравенств, которые получатся после раскрытия модуля. После раскрытия модуля и выполнения необходимых тождественных преобразований, получим систему неравенства:  1 и  −1

неравенство с модулем рис 18

неравенство с модулем рис 19

Решением служит пересечение промежутков  1 и  −1

x ∈ (−∞ ; 1] ∩ [−1 ; +∞)

или покороче:

x ∈ [−1 ; 1]

Обратите внимание, что границы −1 и 1 включены в соответствующие промежутки. Это потому что при подстановке этих чисел в исходное неравенство, получается верное неравенство.

Аналогично решаются неравенства, в левой части которого модуль, а справа не просто число, а буквенное выражение.

Пример 4. Решить неравенство |7 6| < + 12

Решение

Для начала раскроем модуль. Вспоминаем, что если неравенство содержит знак < или ≤, то неравенства получившиеся после раскрытия модуля, следует записать в виде системы. Это будет означать, что итоговые решения будут удовлетворять обоим неравенствам.

Итак, после раскрытия модуля получим следующую систему:

неравенство с модулем рис 20

В данном случае система содержит не совсем элементарные неравенства как в прошлых примерах. Данные неравенства следует упростить, используя известные тождественные преобразования.

Раскроем скобки во втором неравенстве. Тогда получим следующую систему:

неравенство с модулем рис 22

В обоих неравенствах выражения, содержащие неизвестные, перенесём в левую часть, а числовые выражения — в правую. Затем приведём подобные слагаемые. Тогда получим систему:

неравенство с модулем рис 23

В первом неравенстве разделим обе части на 6. Во втором неравенстве разделим обе части на −8. Тогда получим окончательную систему:

неравенство с модулем рис 24

Изобразим решения на координатной прямой:

неравенство с модулем рис 25

Решением является пересечение промежутков (−∞ ; 3) и , то есть промежуток неравенство с модулем рис 30

неравенство с модулем рис 29

Ответ: неравенство с модулем рис 30


Пример 5. Решить неравенство |1 − 2x| ≥ 4 − 5x

Решение

Для начала раскроем модуль. Вспоминаем, что если неравенство содержит знак > или ≥, то неравенства получившиеся после раскрытия модуля, следует записать в виде совокупности:

После раскрытия модуля получим следующую совокупность:

неравенство с модулем рис 31

Выполним необходимые тождественные преобразования в обоих неравенствах. В результате получим:

неравенство с модулем рис 32

Изобразим решения на координатной прямой:

неравенство с модулем рис 33

Решением является объединение промежутков неравенство с модулем рис 34 и [1 ; +∞), то есть промежуток неравенство с модулем рис 34

неравенство с модулем рис 35

Ответ: неравенство с модулем рис 34.

Задания для самостоятельного решения

Задание 1. Решить неравенство:
Решение:
Ответ: x ∈ (−36 ; 36).
Задание 2. Решить неравенство:
Решение:
Ответ: x ∈ (−∞ ; −2) ∪ (2 ; +∞).
Задание 3. Решить неравенство:
Решение:
Ответ:
Задание 4. Решить неравенство:
Решение:
Ответ: x ∈ [−5 ; 2]
Задание 5. Решить неравенство:
Решение:
Ответ: x ∈ (−∞ ; 0)
Задание 6. Решить неравенство:
Решение:
Ответ:

Понравился урок?
Вступай в нашу новую группу Вконтакте и начни получать уведомления о новых уроках

Возникло желание поддержать проект?
Используй кнопку ниже

Решение уравнений с модулем методом интервалов

Уравнения с несколькими модулями в одной части

Чем больше модулей, тем больше приходиться их раскрывать и тем больше получается различных уравнений. Когда модулей один или два — это не сложно. Сложность возникает когда модулей больше двух. Человек может забыть рассмотреть какой-то из случаев, и получится что уравнение решено не полностью.

Давайте решим следующее уравнение:

|− 5| − |x| = 1

У данного уравнения два модуля в левой части. Оно решается путем раскрытия модулей. Не будем комментировать решение, а сразу приведём его:

uravnenie-s-modulem-risunok-52

Такой вид уравнения удобнее решать методом интервалов (или более точно — методом промежутков). Суть этого метода в том, чтобы разбить координатную прямую на несколько промежутков, а затем решить уравнение на каждом из этих промежутков. Модули исходного уравнения на каждом промежутке будут раскрываться по разному.

Решим уравнение | 5|  |x| = 1 методом интервалов.

Для начала нарисуем координатную прямую и обозначим её как x

уравнение с модулем рисунок 49

Если координатная прямая содержит все числа, которые существуют в природе, то логично что она содержит и корни нашего уравнения.

Теперь надо разбить координатную прямую на промежутки. Для этого сначала нужно найти на ней те точки, на которых модули нашего уравнения будут менять свой порядок раскрытия. То есть, найти точки перехода для модулей |− 5| и |x|.

Чтобы найти точки перехода, нужно выяснить при каких значениях x подмодульные выражения равны нулю. Узнать это можно приравняв к нулю подмодульные выражения обоих модулей, и решить обычные линейные уравнения:

метод интервалов рис 1

Для модуля |− 5| точкой перехода будет 5. Для модуля |x| точкой перехода будет 0.

Теперь отметим точки перехода на координатной прямой. Мéньшие числа нужно отмечать левее, большие числа правее:

уравнение с модулем рисунок 51

Проведем дуги от точек перехода:

уравнение с модулем рисунок 52

С помощью неравенств подпишем каждый промежуток. Получится три промежутка: от минус бесконечности до нуля, от нуля до пяти, и от пяти до плюс бесконечности. То есть: x < 0, 0 ≤ x < 5 и x ≥ 5

уравнение с модулем рисунок 53

Обратите внимание, что в первом промежутке x < 0 значение 0 не включено в данный промежуток. Но зато это значение включено во второй промежуток 0 ≤ < 5.

Во втором же промежутке 0 ≤ x < 5 значение 5 не включено в данный промежуток, но зато оно включено в третий промежуток x ≥ 5.

Проще говоря, каждый промежуток включает в себя левый конец, и не включает правый. Сделано это специально, чтобы не допустить потерь значений переменной x. Описать с помощью неравенств нужно все значения на координатной прямой, не допуская их потерь.

уравнение с модулем рисунок 53

Включение левого конца в рассматриваемый промежуток и исключение его из правого это лишь общепринятое правило. На самом деле концы рассматриваемого промежутка можно включать в любой из соседствующих промежутков. Например, значение 0 можно было включить в первый промежуток. Тогда он принял бы вид ≤ 0, а второй промежуток принял бы вид 0 < < 5, потому что ноль уже был включен в первый промежуток.

Но лучше всего исходить из ситуации, потому что в каких-то случаях левый конец промежутка целесообразнее исключить из рассматриваемого промежутка и включить его в правый конец соседнего промежутка. Об этом мы поговорим позже.

Теперь выясним как будут вести себя модули |− 5| и |x| на каждом из этих промежутков. От этого будет зависеть то, как они будут раскрываться.

Начнем с первого промежутка x < 0.

Если x < 0, то при любом значении x на данном промежутке подмодульное выражение − 5 станет отрицательным, а значит модуль |− 5| на промежутке x < 0 будет раскрываться со знаком минус. Второй модуль |x| на промежутке x < 0 тоже будет раскрываться со знаком минус.

В результате после раскрытия модулей на промежутке x < 0 уравнение с модулем |x − 5| − |x| = 1 примет вид −(− 5) + x = 1

уравнение с модулем рисунок 55

Второй модуль |x| на промежутке < 0 раскрылся с минусом. В самом же уравнении |− 5 |− |x| = 1 после выражения |x − 5| тоже располагался минус. В математике два минуса, идущие подряд, дают плюс. Поэтому и получилось выражение −(− 5) + x = 1.

Решим уравнение −( 5) + x = 1, которое получилось после раскрытия модулей на промежутке x < 0

уравнение с модулем рисунок 59

Это уравнение решений не имеет. Значит на промежутке < 0 исходное уравнение не имеет корней. Проще говоря, корень уравнения не является числом меньшим нуля.

Следующий промежуток, на котором нужно решить уравнение это промежуток 0  < 5.

Если x больше или равно нулю, но меньше пяти, то подмодульное выражение  5, станет отрицательным, а значит модуль | 5| на промежутке 0  x < 5 будет раскрываться со знаком минус. Второй модуль |x| на промежутке 0  < 5 будет раскрываться с плюсом.

В результате после раскрытия модулей на промежутке 0  x < 5 уравнение с модулем |x − 5| − |x| = 1 примет вид −( 5) − x = 1

уравнение с модулем рисунок 56

Решим это уравнение:

уравнение с модулем рисунок 60

Получили корень 2. Чтобы проверить действительно ли это число является  корнем исходного уравнения, нужно посмотреть принадлежит ли это число рассматриваемому промежутку  0  x < 5. Принадлежит? Да. Значит число 2 является корнем уравнения |x − 5| − |x| = 1. Проверка также показывает это:

уравнение с модулем рисунок 70

Следующий промежуток, который нужно рассмотреть это промежуток x ≥ 5.

Если x больше или равно пяти, то модуль |− 5| на промежутке x ≥ 5 будет раскрываться со знаком плюс. Второй модуль |x| на промежутке x ≥ 5 тоже будет раскрываться с плюсом.

В результате после раскрытия модулей на промежутке x ≥ 5 уравнение с модулем |x − 5| − |x| = 1 примет вид − 5 − x = 1.

уравнение с модулем рисунок 57

Решим это уравнение:

уравнение с модулем рисунок 71

Это уравнение не имеет решений. Значит на промежутке x ≥ 5 исходное уравнение корней не имеет. Проще говоря, корень уравнения не является числом, бóльшим либо равным пяти.

В итоге корнем уравнения является число 2, которое мы нашли решив исходное уравнение на промежутке 0  x < 5.

Ответ: 2.


Пример 2. Решить уравнение |− 3| + |+ 2| = 7

Решение

Шаг 1. Находим точки перехода для модулей |− 3| и |+ 2|

метод интервалов рис 2

Шаг 2. Отметим на координатной прямой найденные точки перехода и выделим получившиеся промежутки:

метод интервалов рис 3

Шаг 3. Решим исходное уравнение на каждом промежутке. Для этого посмóтрим как будут раскрываться модули | 3| и |+ 2| на этих промежутках.

На промежутке < −2 модуль |− 3| будет раскрываться с минусом. Можно проверить это, подставив в данный модуль любое число из промежутка < −2. Например, числа −4 или −9

|− 3| = |−4 − 3| = |−7| = −(−7) = 7

|− 3| = |−9 − 3| =|−12| = −(−12) = 12

Следующий модуль |+ 2| на промежутке < −2 тоже будет раскрываться с минусом. Убедимся в этом подставив любые два числа из промежутка < −2 в подмодульное выражение. Например, числа −6 и −8

|+ 2| = |−6 + 2| = |−4| = −(−4) = 4

|+ 2| = |−8 + 2| = |−6| = −(−6) = 6

Значит после раскрытия модулей на промежутке < −2 исходное уравнение | 3| + |+ 2| = 7 принимает следующий вид:

+ 3   2 = 7

Решим его:

метод интервалов рис 8

Обязательно нужно проверить входит ли найденный корень −3 в рассматриваемый промежуток < −2. Для этого нужно подставить в неравенство < −2 найденный корень −3 и проверить верное ли оно. В данном случае неравенство −3 < −2 верно, значит корень −3 входит в промежуток < −2 и соответственно является корнем исходного уравнения.

На следующем промежутке −2  < 3 модуль | 3| будет раскрываться с минусом, а модуль|+ 2| будет раскрываться с плюсом.

Значит после раскрытия модулей на промежутке −2  < 3 исходное уравнение | 3| + |+ 2| = 7 принимает следующий вид:

+ 3 + + 2 = 7

Решим это уравнение:

метод интервалов рис 5

Это уравнение не имеет решений, значит на промежутке −2 ≤ < 3  исходное уравнение тоже не имеет решений (корней).

Наконец рассмотрим промежуток  3

На промежутке  3 модуль | 3| будет раскрываться с плюсом. Модуль|+ 2| так же будет раскрываться с плюсом. Значит на промежутке ≥ 3 исходное уравнение | 3| + |+ 2| = 7 принимает следующий вид:

x − 3 + + 2 = 7

Решим это уравнение:

метод интервалов рис 6

Этот корень входит в рассматриваемый промежуток ≥ 3, значит является корнем исходного уравнения. Проверка также показывает это:

метод интервалов рис 7

Ответ: −3 и 4.


Пример 3. Решить уравнение |2 3| + |2+ 7| = 16

Решение

Найдём точки перехода для модулей |2x  3| и |2x + 7|

метод интервалов рис 9

Отметим точки перехода на координатной прямой. Меньшие числа нужно отмечать левее, большие правее:

метод интервалов рис 10

Решим исходное уравнение |2 3| + |2+ 7| = 16 на промежутке метод интервалов рис 11. Оба модуля на этом промежутке будут раскрываться с минусом:

метод интервалов рис 12

Корень −5 принадлежит промежутку метод интервалов рис 11, значит является корнем исходного уравнения.

Теперь решим исходное уравнение на промежутке метод интервалов рис 13. Модуль |2x  3| на этом промежутке раскрывается с минусом, а модуль |2x + 7| — с плюсом:

метод интервалов рис 14

Видим, что на промежутке исходное уравнение не имеет решений (корней).

Теперь решим исходное уравнение на промежутке метод интервалов рис 15. Оба модуля на данном промежутке раскрываются с плюсом:

метод интервалов рис 16

Корень 3 принадлежит промежутку метод интервалов рис 15, значит является корнем исходного уравнения.

Ответ: −5 и 3.


Пример 4. Решить уравнение | 2| + 3= | 5|  18

Решение

Найдём точки перехода для модулей |x  2| и |x 5|

метод интервалов рис 17

Отметим точки перехода на координатной прямой:

метод интервалов рис 18

Решим исходное уравнение на промежутке < 2. Модули |− 2| и |− 5| на этом промежутке раскрываются с минусом:

метод интервалов рис 19

Число −5 принадлежит промежутку < 2, значит является корнем исходного уравнения.

Решим исходное уравнение на промежутке 2  < 5. Модуль | 2| на этом промежутке раскрывается с плюсом, а модуль |− 5| — с минусом:

метод интервалов рис 20

Число -11 na 5 не принадлежит промежутку 2  x < 5, значит не является корнем исходного уравнения.

Решим исходное уравнение на промежутке  5. Модули |− 2| и |− 5| на этом промежутке будут раскрываться с плюсом:

метод интервалов рис 21

Число −7 не принадлежит промежутку  5, значит не является корнем исходного уравнения.

Ответ: −5


Пример 5. Решить уравнение |x| + |x − 7| + 2| 4| = 2

Решение

Найдём точки перехода для модулей |x|, |x − 7| и |x 4|

метод интервалов рис 22

Отметим точки перехода на координатной прямой:

метод интервалов рис 23

Решим исходное уравнение на промежутке < 0. Все три модуля: |x|, |− 7| и |x 4| на этом промежутке раскрываются с минусом:

метод интервалов рис 24

Число 13 na 4 не принадлежит промежутку < 0, значит не является корнем исходного уравнения.

Решим теперь исходное уравнение на промежутке 0  < 4. Модуль |x| на этом промежутке раскрывается с плюсом, а модули |− 7| и |x 4| — с минусом:

метод интервалов рис 25

Число метод интервалов рис 26 не принадлежит промежутку 0  < 4, значит не является корнем исходного уравнения.

Решим теперь исходное уравнение на промежутке 4  < 7. Модуль |x| на этом промежутке раскрывается с плюсом; модуль | 7| — с минусом; модуль |− 4| — с плюсом:

метод интервалов рис 27

Число три вторых не принадлежит промежутку 4  < 7, значит не является корнем исходного уравнения.

Решим исходное уравнение на промежутке x ≥ 7. Все три модуля: |x|, |x − 7| и |x 4| на этом промежутке раскрываются с плюсом:

метод интервалов рис 28

Число 17 na 4 не принадлежит промежутку x ≥ 7, значит не является корнем исходного уравнения.

Решив исходное уравнение на каждом промежутке, мы не нашли корней, удовлетворяющих этому уравнению. Значит данное уравнение не имеет корней.

В ответе можно написать словами, что корней нет (или решений нет), либо указать символ пустого множества. Этот символ будет указывать, что множество корней уравнения |x| + |x − 7| + 2| 4| = 2 пусто.

Ответ: ø.


Пример 6. Решить уравнение метод интервалов рис 29

Решение

Найдём точки перехода для модулей метод интервалов рис 30 и метод интервалов рис 31

Если методом интервалов нужно решить уравнение с модулем, который в свою очередь содержит внутри себя другой модуль, то точки перехода надо искать для случаев: когда внутренний модуль раскрывается с плюсом и когда он раскрывается с минусом. Точки перехода будут меняться в зависимости от этих случаев. Давайте посмотрим как это происходит.

Если у модуля метод интервалов рис 30 внутренний модуль раскроется с плюсом, то есть если 2− 1 ≥ 0 (что равносильно метод интервалов рисунок 40), то исходное уравнение примет вид |2− 1 − 5| + = |6 − x|. Здесь и далее надо учесть, что внутренний модуль будет раскрываться с плюсом при тех значениях x, которые будут больше либо равны одна вторая. Отметим эту точку на координатной прямой.

метод интервалов рис 39

Теперь найдем точки перехода. Поскольку исходное уравнение приняло вид |2− 1 − 5| + = |6 − x|, то точки перехода надо найти для модулей |2− 1 − 5| и |6 − x|.

Для модуля |2− 1 − 5| точкой перехода будет число 3, а для модуля |6 − x| — число 6. Отметим эти числа на той же координатной прямой где мы отметили точку одна вторая

метод интервалов рис 40

Сейчас нас интересуют только те значения x, которые удовлетворяют условию метод интервалов рис 42, потому что только при этом условии внутренний модуль исходного уравнения раскрывается с плюсом. Поэтому рассматривать промежуток метод интервалов рис 41 мы не будем. Рассмотреть нужно те промежутки где x удовлетворяет условию метод интервалов рис 42

метод интервалов рис 43

Первый промежуток на котором мы будем решать уравнение это метод интервалов рис 44. На нем модуль |2 1  5| раскрывается с минусом, а модуль |6  x| с плюсом:

метод интервалов рис 45

Получили тождество — равенство верное при любом значении x. В данном случае решением исходного уравнения является любое число из промежутка метод интервалов рис 44. Любое число из этого промежутка также удовлетворяют условию метод интервалов рис 42

Теперь решим исходное уравнение на промежутке 3  < 6. Оба модуля на этом промежутке раскрываются с плюсом. Тогда:

метод интервалов рис 46

Корень 3 принадлежит рассматриваемому промежутку. Также этот корень удовлетворяет условию метод интервалов рис 42, согласно которому внутренний модуль исходного уравнения раскрывается с плюсом.

Теперь решим исходное уравнение на промежутке  6. На этом промежутке модуль |2 1  5| раскрывается с плюсом, а модуль |6  x| с минусом. Тогда:

метод интервалов рис 47

Корень 0 не удовлетворяет условию  6, значит на данном промежутке исходное уравнение корней не имеет.

Итак, если внутренний модуль уравнения метод интервалов рис 29 раскрывается с плюсом, то решениями уравнения являются: промежуток метод интервалов рис 44, а также число 3. Запишем эти решения одним промежутком:

метод интервалов рис 48

Теперь решим исходное уравнение для случая когда внутренний модуль раскрывается с минусом. То есть когда 2− 1 < 0 (что равносильно неравенству метод интервалов рисунок 41). В этом случае исходное уравнение примет вид:

|−2x + 1 − 5| + x = |6 − x|

Отметим точку одна вторая на координатной прямой.

метод интервалов рис 49

Нас будут интересовать те значения x которые располагаются слева от одна вторая. Это те значения при которых внутренний модуль исходного уравнения раскрывается с минусом.

Найдем точки перехода для модулей |−2+ 1  5| и |6  x|. Для первого модуля это число −2, для второго модуля — число 6

метод интервалов рис 50

Рассматривать будем только те промежутки, которые располагаются слева от одна вторая. Только при них внутренний модуль исходного уравнения раскрывается с минусом

метод интервалов рис 51

Решим уравнение на промежутке < −2. На этом промежутке оба модуля раскрываются с плюсом. Тогда:

метод интервалов рис 52

Это уравнение решений не имеет. Значит на промежутке < −2 исходное уравнение не имеет корней.

Решим теперь уравнение на промежутке метод интервалов рисунок 42. Замечаем, что при подстановке левого конца этого промежутка (числа −2) в модуль |−2+ 1  5| данный модуль раскрывается с плюсом, а при остальных значениях промежутка метод интервалов рисунок 42 модуль |−2+ 1 − 5| раскрывается с минусом.

Поэтому число −2 разумнее включить в промежуток < −2, который мы уже рассмотрели. На промежутке < −2 модуль раскрывался с плюсом, и при включении числа −2 в данный промежуток, он также будет раскрываться с плюсом.

На промежутке метод интервалов рисунок 42 модуль |−2+ 1 − 5| раскрывается с минусом, а модуль |6 − x| с плюсом. Тогда:

метод интервалов рис 53

Получится корень который не удовлетворяет условию метод интервалов рисунок 42. Несмотря на это число одна вторая является корнем исходного уравнения, потому что мы получили его когда решали уравнение для случая 2− 1 ≥ 0.


Задания для самостоятельного решения

Примечание: Решения, не удовлетворяющие исходному уравнению, подчёркнуты красным.

Задание 1. Решить уравнение:
Решение:
Ответ: x ∈ [−5 ; 3].
Задание 2. Решить уравнение:
Решение:
Ответ: x ∈ [3 ; +∞).
Задание 3. Решить уравнение:
Решение:
Ответ: корней нет.
Задание 4. Решить уравнение:
Решение:
Ответ: , 0.
Задание 5. Решить уравнение:
Решение:
Ответ: −5.
Задание 6. Решить уравнение:
Решение:
Ответ: −4, 2.
Задание 7. Решить уравнение:
Решение:
Ответ: , .

Понравился урок?
Вступай в нашу новую группу Вконтакте и начни получать уведомления о новых уроках

Возникло желание поддержать проект?
Используй кнопку ниже

Уравнение с модулем

Уравнение с модулем достаточно сложная тема для начинающих. Учитывая это обстоятельство, в данный урок войдут только элементарные уравнения.

Что такое уравнение с модулем и как его решить?

В уравнениях с модулем неизвестное значение содержится под знáком модуля. Например:

|x − 2| = 5

Уравнения с модулем бывают разными и решаются они различными методами. Нельзя сказать что какой-то метод наиболее рационален. Всё зависит от исходного уравнения.

Например, в каких-то уравнениях можно просто угадать корень, в то время как в других нужно логически мыслить, раскрывать модули, выполнять тождественные преобразования. Человек волен выбирать каким методом решения пользоваться.

К примеру, решим вышеприведённое уравнение |x − 2| = 5. Допустим, что мы не знаем ни одного метода решения. Как бы мы его решили?

Прежде всего заметим, что правая часть данного уравнения равна числу 5. Слева же располагается модуль из выражения |x − 2|. Это означает что подмодульное выражение x − 2 должно равняться числу 5 или −5

модуль числа рисунок 30

Значит нужно выяснить при каких значениях переменной x подмодульное выражение x − 2 будет обращаться в число 5 или −5.

Искомые значения x найдутся если приравнять подмодульное выражение к числу 5 и −5, а затем поочерёдно решить каждое из уравнений:

уравнение с модулем рисунок 11

Значит корнями уравнения |x − 2| = 5 являются числа 7 и −3.

Большинство элементарных уравнений с модулем можно решить используя правило раскрытия модуля. Для этого раскрывают модуль содержащийся в уравнении, затем получившееся выражение подставляют в исходное уравнение вместо выражения с модулем.

Раскрывать модуль нужно для каждого из случаев: когда подмодульное выражение больше или равно нулю, и когда подмодульное выражение меньше нуля.

Решим наше уравнение |x − 2| = 5 с помощью правила раскрытия модуля. Выпишем отдельно его модуль и раскроем его:

модуль числа рисунок 29

В этой конструкции говорится, что если подмодульное выражение − 2 больше или равно нулю, то модуль раскроется как − 2, и тогда исходное уравнение примет вид − 2 = 5, откуда = 7

модуль числа рисунок 31

А если же подмодульное выражение − 2 меньше нуля, то модуль раскроется как −(− 2). Тогда исходное уравнение примет вид −(− 2) = 5, откуда = −3

модуль числа рисунок 32

Итак, уравнение |x − 2|= 5 имеет корни 7 и −3. Для проверки подстáвим числа 7 и −3 в исходное уравнение вместо x. Тогда получим верное равенство:

модуль числа рисунок 22

Подмодульное выражение как правило содержит такое x, которое может обращать всё подмодульное выражение как в положительное число, так и в отрицательное, либо вообще в ноль.

Поэтому модуль и раскрывается для каждого из случаев: когда подмодульное выражение больше или равно нулю, и когда подмодульное выражение меньше нуля. Каждый из случаев будет давать независимое уравнение со своим корнем.

Вернёмся теперь к моменту, где мы раскрывали модуль:

модуль числа рисунок 29

Условия − 2 ≥ 0 и − 2 < 0 являются неравенствами, которые можно решить, тем самым приведя их к простому виду:

уравнение с модулем рисунок 46

Символ означает равносильность. В данном случае указывается, что условие  2  0 равносильно условию  2, а условие  2 < 0 равносильно условию < 2.

Такой вид записи условий позволяет однозначно сказать при каких x модуль будет раскрываться с плюсом, а при каких с минусом.

В первом случае получилось условие ≥ 2. Это значит что при всех x бóльших либо равных 2, модуль |− 2| будет раскрываться с плюсом. Так, при = 7, подмодульное выражение станет равно 5

|7 − 2| = |5|

А значит дальнейшее раскрытие будет с плюсом

|7 − 2| = |5| = 5

Таким же образом модуль |− 2| будет вести себя и с другими значениями x на промежутке ≥ 2. То есть, будет раскрываться с плюсом. Примеры:

При x = 3, |3 − 2|=|1| = 1
При = 4, |4 − 2|=|2| = 2
При = 2, |2 − 2|=|0| = 0
При = 13, |13 − 2|=|11| = 11

А во втором случае получилось условие < 2. Это значит что при всех x мéньших 2, модуль будет раскрываться с минусом. Так, при = −3, подмодульное выражение опять же станет равно 5. Но в промежуточных вычислениях можно увидеть, что модуль раскрывается с минусом:

|−3 − 2| = |−5| = −(−5) = 5

Модуль |− 2| будет вести себя так же и с другими значениями x на промежутке x < 2. Примеры:

При = 1, |1 − 2|=|−1| = −(−1) = 1
При = 0, |0 − 2|=|−2| = −(−2) = 2
При = −1, |−1 − 2|=|−3| = −(−3) = 3
При = −9,|−9 − 2|=|−11| = −(−11) = 11

Число 2 является своего рода точкой перехода, в которой модуль |− 2| меняет свой порядок раскрытия.

Можно представить как модуль |− 2| двигался по маршруту от минус бесконечности до числа 2, раскрываясь в каждой точке с минусом. Попав в точку 2, модуль поменял свой порядок раскрытия — а именно раскрывшись в точке 2 с плюсом, он далее стал раскрываться с плюсом, двигаясь в правую часть к плюс бесконечности.

С помощью координатной прямой это можно представить так:

уравнение с модулем рисунок 48

Красные знаки минуса и плюса указывают, как будет раскрываться модуль |− 2| на промежутках < 2 и ≥ 2.

Точку перехода можно найти для любого модуля. Для этого нужно узнать при каких x подмодульное выражение равно нулю. Ноль это то значение, до и после которого модуль всегда сохраняет свой знак. Это следует из правила раскрытия модуля:

модуль числа свойство

В этом примере в момент когда x станет равным нулю, модуль |x| раскроется с плюсом и далее при всех x, бóльших нуля, будет раскрываться с плюсом. Напротив, при всех x, мéньших нуля модуль будет раскрываться с минусом:

модуль рисунок 1

А например для модуля |2+ 6| точкой перехода будет число −3, потому что при его подстановке в подмодульное выражение 2+ 6 вместо x, данное подмодульное выражение станет равно нулю. Изобразим это на рисунке:

уравнение с модулем рисунок 50

При всех x, бóльших либо равных −3, модуль будет раскрываться с плюсом. Примеры:

При = −3, |2 × (−3) + 6| = |0| = 0
При = 4, |2 × 4 + 6| = |14| = 14
При = 5, |2 × 5 + 6| = |16| = 16

А при всех x, мéньших 3, модуль будет раскрываться с минусом. Примеры:

При = −4, |2 × (−4) + 6| = |−2| = −(−2) = 2
При = −5, |2 × (−5) + 6| = |−4| = −(−4) = 4
При = −6, |2 × (−6) + 6| = |−6| = −(−6) = 6


Пример 2. Решить уравнение |x| + 3= −2

Решение

Раскроем модуль, который содержится в левой части уравнения:

модуль числа свойство

Если x ≥ 0, то модуль раскроется со знаком плюс и тогда исходное уравнение примет вид x + 3x = −2. Сразу решим это уравнение:

уравнение с модулем рисунок 40

Теперь рассмотрим второй случай — когда x < 0. В этом случае модуль в исходном уравнении раскроется со знаком минус, и тогда получится уравнение x + 3x = −2. Решим и это уравнение:

уравнение с модулем рисунок 41

Получили корни минус одна вторая и −1.

Выполним проверку, подставив найденные корни в исходное уравнение. Проверим корень минус одна вторая

уравнение с модулем рисунок 42

Видим, что при подстановке корня минус одна вторая исходное уравнение не обращается в верное равенство. Значит минус одна вторая не является корнем исходного уравнения.

Проверим теперь корень −1

уравнение с модулем рисунок 43

Получили верное равенство. Значит из двух найденных решений только −1 является корнем уравнения.

Ответ: −1.

Здесь можно сделать важный вывод. В уравнениях с модулем найденные корни не всегда удовлетворяют исходному уравнению. Чтобы убедиться в правильности своего решения, нужно выполнять проверку, подставляя найденные корни в исходное уравнение.

Кроме того, проверить является ли найденное значение корнем уравнения можно с помощью условия, согласно которому был раскрыт модуль.

Так, в данном примере мы раскрывали модуль |x| для случаев когда подмодульное выражение больше или равно нулю, и когда подмодульное выражение меньше нуля:

уравнение с модулем рисунок 93

Условия x≥0 и x<0 являются неравенствами. В эти неравенства можно подставлять найденные корни. Если неравенства окажутся верными, значит корни удовлетворяют исходному уравнению.

Так, при раскрытии модуля со знаком плюс, получилось уравнение + 3= −2. Корнем этого уравнения стало число минус одна вторая. Это число не удовлетворяет условию ≥ 0, согласно которому был раскрыт модуль |x| и согласно которому было получено уравнение x + 3x = −2. Действительно, при подстановке числа минус одна вторая в неравенство ≥ 0 получается неверное неравенство.

А при раскрытии модуля со знаком минус, получилось уравнение x + 3x = −2. Корнем этого уравнения стало число −1. Это число удовлетворяет условию x<0, согласно которому был раскрыт модуль |x| и согласно которому было получено уравнение x + 3x = −2. Действительно, при подстановке числа −1 в неравенство x < 0 получается верное неравенство.


Пример 3. Решить уравнение |1 − 2x| − 4= −6

Решение

Раскроем модуль:

уравнение с модулем рисунок 80

При раскрытии модуля |1 − 2x| со знаком плюс, получим уравнение 1 − 2x − 4= −6. Решим его:

уравнение с модулем рисунок 81

При раскрытии модуля |1 − 2x| со знаком минус, получим уравнение −1 + 2− 4= −6. Решим его:

уравнение с модулем рисунок 83

Получили корни семь шестых и пять вторых.

Корень семь шестых не удовлетворяет условию уравнение с модулем рисунок 82, значит не является корнем исходного уравнения.

Корень пять вторых удовлетворяет условию уравнение с модулем рисунок 84, значит является корнем исходного уравнения. Проверка также покажет это:

уравнение с модулем рисунок 85

Ответ: пять вторых.


Пример 4. Решить уравнение |x− 3x| = 0

Решение

Если модуль числа равен нулю, то подмодульное выражение тоже равно нулю:

уравнение с модулем рисунок 86

То есть можно не раскрывать модуль. Достаточно узнать при каких значениях x подмодульное выражение равно нулю. В данном случае для этого нужно решить неполное квадратное уравнение:

уравнение с модулем рисунок 87

Получили корни 0 и 3. Оба корня удовлетворяют исходному уравнению. Проверка показывает это:

уравнение с модулем рисунок 88


Пример 5. Решить уравнение x− 5|x| + 6 = 0

Выпишем отдельно модуль |x| и раскроем его:

модуль числа свойство

При раскрытии модуля |x| со знаком плюс, исходное уравнение примет вид x− 5+ 6 = 0. Это квадратное уравнение. Решим его с помощью дискриминанта:

уравнение с модулем рисунок 94

Оба корня удовлетворяют условию ≥ 0, значит являются корнями исходного уравнения.

При раскрытии модуля |x| со знаком минус, исходное уравнение примет вид x2 + 5+ 6 = 0. Это тоже квадратное уравнение. Решим его как и предыдущее:

уравнение с модулем рисунок 95

При условии ≥ 0, модуль из уравнения раскрылся с плюсом, получились корни 3 и 2. Оба корня удовлетворяют условию ≥ 0, значит удовлетворяют и исходному уравнению.

При условии < 0, модуль из уравнения раскрылся с минусом, получились корни −2 и −3. Оба корня удовлетворяют условию < 0, значит удовлетворяют и исходному уравнению.

Ответ: 3, 2, −2 и −3.


Сведéние уравнения с модулем в совокупность

Большинство элементарных уравнений с модулем можно решить сведéнием их к так называемой совокупности уравнений.

Элементарными мы будем называть те уравнения с модулем, в которых левая часть является модулем из какого-то выражения, а правая часть — числом. Например, |x| = 3  или |2− 1| = 3.

Решим наше самое первое уравнение |− 2| = 5 сведéнием его к совокупности уравнений. Корнями этого уравнения были числа 7 и −3. Это уравнение тоже считается элементарным.

Если раскрыть модуль |− 2| со знаком плюс, то уравнение |− 2| = 5 примет вид − 2 = 5.

Если раскрыть модуль |− 2| со знаком минус, то уравнение |− 2| = 5 примет вид −(− 2) = 5, то есть+ 2 = 5.

Видим, что из уравнения |− 2| = 5 получилось два уравнения: − 2 = 5 и + 2 = 5. Причём каждое из уравнений имеет свой собственный корень. Уравнение − 2 = 5 имеет корень 7, а уравнение + 2 = 5 — корень −3

Выпишем уравнения x − 2 = 5 и x + 2 = 5 и объединим их квадратной скобкой:

модуль числа рисунок 39

Такой вид записи называют совокупностью уравнений.

Совокупность уравнений — это несколько уравнений, объединённых квадратной скобкой, и имеющих множество решений, которые удовлетворяют хотя бы одному из уравнений, входящих в данную совокупность.

Так, число 7 является решением совокупности модуль числа рисунок 39 потому что это число удовлетворяет первому уравнению х − 2 = 5.

Число −3 тоже является решением данной совокупности, поскольку удовлетворяет второму уравнению −х + 2 = 5.

Вместе же числа 7 и −3 образуют множество решений данной совокупности.

В отличие от системы уравнений, совокупность состоит из уравнений, которые не зависят друг от друга. Для каждого уравнения, входящего в совокупность, значение переменной x будет разным. А в системе уравнений значение переменной x удовлетворяет как первому уравнению, так и второму.

Решить совокупность уравнений означает найти множество решений, которые удовлетворяют хотя бы одному из уравнений, входящих в данную совокупность.

Решим каждое уравнение совокупности модуль числа рисунок 39 по-отдельности. Это обычные линейные уравнения, которые легко решаются:

уравнение с модулем рисунок 3

Символ ⇔ как было ранее сказано означает равносильность. В данном случае он указывает на то, что все получающиеся совокупности равносильны друг другу.

Итак, мы получили корни 7 и −3. Поскольку эти два числа являются решениями совокупности модуль числа рисунок 39, то значит являются и решениями уравнения |x − 2| = 5.

В исходную совокупность можно включать условия, согласно которым был раскрыт модуль. В этом случае каждое уравнение вместе со своим условием обрамляется знаком системы.

Дополним предыдущую совокупность условиями, согласно которым был раскрыт модуль. К первому уравнению x − 2 = 5 добавим условие x − 2 ≥ 0, а ко второму уравнению −x + 2 = 5 добавим условие x − 2 < 0

модуль числа рисунок 44

Решение каждого уравнения должно удовлетворять своему условию. Поэтому условия и уравнения обрамлены знáком системы.

Решим получившуюся совокупность с условиями. Условия являются неравенствами, которые тоже можно решать:

уравнение с модулем рисунок 4

В первом случае получили корень 7, который удовлетворяет своему условию x ≥ 2. Во втором случае получили корень −3, который удовлетворяет своему условию x < 2.

Не следует бояться таких записей. Это лишь подробное решение, показывающее что откуда взялось. Чаще всего решение можно записать покороче.

Существует схема для сведéния в совокупность уравнения вида |x| = a. Выглядит эта схема так:

уравнение с модулем рисунок 22

Данная схема легко позволяет свести уравнение с модулем в совокупность. Эту схему можно прочитать так: «Если выражение |x| равно a, то подмодульное выражение равно a или −a»

Квадратная скобка в совокупностях заменяет собой слово «или».

Например, уравнение |x| = 5 можно свести в совокупность, рассуждая так: если выражение |x| равно 5, то подмодульное выражение равно 5 или −5.

уравнение с модулем рисунок 25

А применительно к нашему предыдущему примеру можно рассуждать так: если |− 2| равно 5, то подмодульное выражение равно 5 или −5

уравнение с модулем рисунок 26

Это та же самая совокупность, что и в прошлый раз. Убедитесь в этом, умножив обе части второго уравнения на −1.

В уравнениях где слева модуль, а справа число, мы будем чаще использовать именно такой способ записи совокупности. Он позволяет не прибегать к правилу раскрытия модуля, а сразу получить совокупность.

Но надо помнить, что эта схема будет работать только для уравнений вида |x| = a. То есть для уравнений, у которого слева модуль, а справа число.


Пример 2. Решить уравнение |2− 1| = 3

Решение

У этого уравнения слева модуль, а справа число. Значит его можно свести в совокупность, воспользовавшись схемой уравнение с модулем рисунок 89

Если выражение |2− 1| равно 3, то подмодульное выражение 2− 1 равно 3 или −3

уравнение с модулем рисунок 23

Теперь решим каждое уравнение совокупности по отдельности:

уравнение с модулем рисунок 24

Ответ: 2 и −1.


Пример 3. Решить уравнение |+ 2| − 3 = 8

Решение

В некоторых случаях прежде чем свести исходное уравнение в совокупность, его следует упростить.

Так, в данном случае −3 следует перенести в правую часть, изменив знак:

уравнение с модулем рисунок 8

Получили уравнение |+ 2| = 11. Если выражение |+ 2| равно 11, то подмодульное выражение + 2 равно 11 или −11

уравнение с модулем рисунок 39

Решим данную совокупность:

уравнение с модулем рисунок 27

Ответ: 9 и −13.


Пример 4. Решить уравнение  4|x| + 4 = 2|x| + 10

Решение

Перенесём 2|x| из правой части в левую часть, а 4 перенесём из левой части в правую часть:

4|x| − 2|x| = 10 − 4
2|x| = 6

Разделим обе части получившегося уравнения на 2. Тогда получится простое уравнение с модулем:

уравнение с модулем рис 39

Ответ: 3 и −3.


Пример 5. Решить уравнение уравнение с модулем рисунок 76

Решение

Если выражение |2 − 5x2| равно 3, то подмодульное выражение 2 − 5x2 равно 3 или −3

уравнение с модулем рисунок 77

В обоих уравнениях перенесём 2 в правую часть, изменив знак:

уравнение с модулем рисунок 78

В первом уравнении разделим обе части на −5. Во втором уравнении так же разделим обе части на −5. Тогда получим два квадратных уравнения

Первое уравнение не имеет корней, потому что квадрат любого числа положителен, а в данном случае он равен отрицательному числу. Корнями второго уравнения являются числа 1 и −1, поскольку вторая степень этих чисел равна единице.

Ответ: 1 и −1.


Пример 6. Решить уравнение |+ 6| + 4= 5

Решение

Данное уравнение не является уравнением вида |x| = a, значит не получится воспользоваться схемой уравнение с модулем рисунок 89.

Чтобы свести данное уравнение в совокупность, нужно сначала раскрыть его модуль, затем записать совокупность из получившихся уравнения.

Раскроем модуль |+ 6|

уравнение с модулем рисунок 92

Если + 6 ≥ 0 , то модуль раскроется со знаком плюс и тогда исходное уравнение примет вид + 6 + 4= 5

Если + 6 < 0, то модуль раскроется со знаком минус и тогда исходное уравнение примет вид −x − 6 + 4= 5. Получим следующую совокупность:

уравнение с модулем рисунок 90

Дальнейшее решение элементарно:

уравнение с модулем рисунок 91

Из найденных корней только -1 na 5 является корнем исходного уравнения, поскольку удовлетворяет условию ≥ −6. А корень 11 na 3 не является корнем уравнения, поскольку не удовлетворяет условию x < −6.

Ответ: -1 na 5


Наиболее простой вид

Наиболее простой вид уравнения с модулем выглядит так:

| x | = a

где x — корень уравнения, a — произвольное число, бóльшее или рáвное нулю. То есть a ≥ 0

Если условие a ≥ 0 не выполнено, то уравнение |x|= a корней не имеет. Это следует из определения модуля. Действительно, модуль всегда неотрицателен.

Приведем несколько примеров уравнений вида |x| = a

Пример 1. Решить уравнение |x| = 2

Решение

В данном случае сразу видно, что корнями являются числа 2 и −2. Ведь если вместо x подставить эти числа, то получим верное равенство: |−2| = 2 и |2| = 2. Решение для этого уравнения можно записать, сведя его в совокупность:

«Если выражение |x| равно 2, то подмодульное выражение x равно 2 или −2«

уравнение с модулем рисунок 5

Ответ: 2 и −2


Пример 2. Решить уравнение |−x| = 4

Решение

Если выражение |−x| равно 4, то подмодульное выражение равно 4 или −4

уравнение с модулем рисунок 28

Умножим оба уравнения на −1

уравнение с модулем рисунок 29

Ответ: −4 и 4.


Пример 3. Решить уравнение |x| = −7

В данном случае корней нет, поскольку модуль всегда неотрицателен. А в данном случае модуль равен отрицательному числу.

Если уравнение с модулем не имеет корней, обычно пишут что x принадлежит пустому множеству:

x ∈ ø

Напомним, что пустым называют множество, не имеющее элементов.


Модуль внутри модуля

Рассмотрим уравнение:

уравнение с модулем рисунок 30

В этом уравнении слева располагается модуль, который в свою очередь содержит внутри себя другой модуль, а справа уравнения располагается число. Такой вид уравнения с модулем можно решить, сведя его в совокупность с помощью схемы, которую мы рассмотрели ранее:

уравнение с модулем рисунок 22

В нашем случае если выражение уравнение с модулем рисунок 31 равно 9, то подмодульное выражение |2 + x| + 3 равно 9 или −9

уравнение с модулем рисунок 12

В получившейся совокупности имеется два уравнения с модулем. Эти уравнения тоже в свою очередь следует свести в совокупность. Но сначала немного упростим эти уравнения. В первом и во втором уравнении перенесем 3 в правую часть, изменив знак. Тогда получим:

уравнение с модулем рисунок 13

Теперь сведём эти уравнения в совокупности. Первое уравнение распадётся на следующую совокупность:

уравнение с модулем рисунок 14

Сразу решим совокупность уравнение с модулем рисунок 15. Первый корень равен 4, второй −8.

уравнение с модулем рисунок 16

Теперь решим второе уравнение |2 + x| = −12. Но замечаем, что его правая часть равна отрицательному числу. Это уравнение не имеет корней, потому что модуль не может равняться отрицательному числу.

Значит уравнение уравнение с модулем рисунок 30 имеет корни 4 и −8. Проверим эти корни, подставив их в исходное уравнение уравнение с модулем рисунок 30

уравнение с модулем рисунок 29

В данном случае оба корня удовлетворяют исходному уравнению.

Ответ: 4 и −8.

Вообще, уравнение с модулем внутри которого содержится другой модуль, тоже решается различными способами. Какой способ использовать зависит от самогó уравнения. Решим например следующее уравнение:

уравнение с модулем рисунок 34

Здесь уже нельзя использовать схему уравнение с модулем рисунок 89 потому что слева располагается не только модуль, но и переменная x. Конечно, переменную x можно перенести в правую часть, и тогда можно будет свести данное уравнение в совокупность:

уравнение с модулем рисунок 99

Но тогда справа появляется переменная x, на которую нужно будет вводить дополнительное ограничение, чтобы правая часть уравнения не стала отрицательной. Такой способ решения мы рассмотрим позже. А пока решим исходное уравнение с помощью правила раскрытия модуля.

Чтобы раскрыть модули данного уравнения нужно сначала определиться где внешний и где внутренний модуль.

В уравнении уравнение с модулем рисунок 99 внешним модулем является полностью левая часть уравнение с модулем рисунок 100, а внутренним модулем — выражение уравнение с модулем рисунок 101

уравнение с модулем рисунок 102

Значение внешнего модуля зависит от внутреннего модуля, и раскрываться внешний модуль будет исходя от результата который получился в результате вычисления его подмодульного содержимого.

Например, если = 3, то внутренний модуль |3  x| примет значение 0, и в результате всё подмодульное выражение внешнего модуля станет равно −2. А это значит что внешний модуль будет раскрываться с минусом.

||3 − x| − x + 1| = ||3 − 3| − 3 + 1| = ||0| − 3 + 1| = |−2| = −(−2) = 2

А если например x = −2, то внутренний модуль |3 − x| примет значение 5, и в результате всё подмодульное выражение внешнего модуля станет равно 8. А это значит что внешний модуль будет раскрываться с плюсом:

||3 − x| − x + 1| = ||3 − (−2)| − (−2) + 1| = ||5| − (−2) + 1| = | 8 |=8

Поэтому решение будем начинать с раскрытия внутреннего модуля.

Если внутренний модуль раскроется с плюсом, то есть если 3 − ≥ 0 (что равносильно неравенству ≤ 3), то исходное уравнение примет вид:

уравнение с модулем рисунок 115

Теперь уравнение имеет только внешний модуль. Решим его раскрыв модуль:

уравнение с модулем рисунок 100

Если −2+ 4  0, то:

уравнение с модулем рисунок 104

Сейчас нас интересуют только те значения x при которых внутренний модуль раскрывается с плюсом, а это произойдет при условии  3. Поэтому для наглядности рядом с найденным корнем указано, что он удовлетворяет условию  3

Решаем далее. Если −2+ 4 < 0, то:

уравнение с модулем рисунок 105

Несмотря на то, что оба найденных корня удовлетворяют уравнению |−2x+4|=6−x, мы исключаем корень 10 na 3 из решений, потому что нас сейчас интересуют только те значения x, при которых внутренний модуль изначального уравнения раскрывается с плюсом. Поэтому рядом с корнем 10 na 3 указано, что он не удовлетворяет условию  3.

Итак, если внутренний модуль раскрывается с плюсом, исходное уравнение принимает вид |−2+ 4| = 6  x и корнем этого уравнения является число −2.

Теперь решим исходное уравнение для случая, когда внутренний модуль раскрывается с минусом, то есть когда 3  < 0 (что равносильно неравенству > 3). Внутренний модуль будет раскрываться с минусом при всех значениях x больших 3.

Если внутренний модуль раскроется с минусом, то исходное уравнение примет вид:

уравнение с модулем рисунок 106

Модуль −2 равен 2. Тогда получаем простейшее линейное уравнение, корень которого равен 4

уравнение с модулем рисунок 107

Получили корень 4, который удовлетворяет условию > 3.

В итоге корнями уравнения являются числа −2 и 4.

Ответ: 2 и 4.


Пример 3. Решить уравнение || 1|  7| = 10

Решение

Слева располагается модуль, а справа число, значит можно применить схему:уравнение с модулем рисунок 22

В данном случае если выражение ||x − 1| − 7| равно 10, то подмодульное выражение |− 1| − 7 равно 10 или 10. Получится совокупность из двух уравнений:

уравнение с модулем рисунок 108

Упростим получившиеся уравнения. Перенесём число −7 в обоих уравнениях в правую часть, изменив знак:

уравнение с модулем рисунок 109

Второе уравнение корней не имеет. Первое уравнение распадется на совокупность уравнение с модулем рисунок 110, корни которой 18 и −16.

уравнение с модулем рисунок 111

Ответ: 18 и −16.

Решим это же уравнение с помощью раскрытия модулей. Начнем с внутреннего модуля.

Если  1  0 (что равносильно  1), то исходное уравнение примет вид:

уравнение с модулем рисунок 114

Решим получившееся уравнение раскрыв модуль:

уравнение с модулем рисунок 112

Далее решаем уравнение для случаев когда  8  0 и  8 < 0

уравнение с модулем рисунок 113

Сейчас нас интересуют те значения, при которых внутренний модуль исходного уравнения раскрывается с плюсом. А это будет при условии, что  1. Этому условию удовлетворяет только значение 18, поэтому мы пометили его зеленой галочкой для наглядности.

Теперь решим исходное уравнение для случая, когда внутренний модуль раскрывается с минусом, то есть когда  1 < 0 (или что равносильно неравенству < 1).

Если  1 < 0, то исходное уравнение примет вид:

уравнение с модулем рисунок 116

Решим получившееся уравнение раскрыв модуль:

уравнение с модулем рисунок 117

Далее решаем уравнение для случаев когда − 6 ≥ 0 и  6 < 0

уравнение с модулем рисунок 118

Из найденных корней только −16 удовлетворяет условию < 1.

В итоге корнями уравнения || 1|  7| = 10 являются числа 18 и −16.

Видно, что с помощью схемы уравнение с модулем рисунок 89  данное уравнение решилось легче и быстрее, чем способом раскрытия модулей.


Слева модуль, а справа выражение с переменной

Решим следующее уравнение с модулем:

|4− 3| = 3x

Здесь так же применима схема:

уравнение с модулем рисунок 22

То есть, если выражение |4− 3| равно 3x, то подмодульное выражение 4− 3 должно равняться 3x или −3x.

уравнение с модулем рисунок 96

Но в исходном уравнении переменная x содержится не только под знáком модуля, но и в правой части. Нам пока неизвестно какое значение примет переменная x. Если x примет отрицательное значение, то правая часть станет полностью отрицательной. В этом случае корней не будет, потому что модуль не может равняться отрицательному числу.

Поэтому, если мы хотим решить данное уравнение, то при сведéнии его в совокупность, дополнительно следует ввести ограничение в виде условия 3≥ 0. Это будет означать, что правая часть уравнения |4 3| = 3x должна быть больше либо равна нулю:

уравнение с модулем рисунок 16

Совокупность и условие обрамлены знаком системы, потому что решения совокупности должны удовлетворять условию 3x ≥ 0.

Итак, решим совокупность. Условие 3x ≥ 0 является неравенством, которое тоже можно решить:

уравнение с модулем рисунок 30

Получившиеся корни можно подставить в условие ≥ 0 и посмотреть выполняется ли оно. Если выполняется, то найденные корни удовлетворяют уравнению. В данном случае при подстановке обеих корней в неравенство, оно выполняется. Проверка также показывает, что корни удовлетворяют уравнению:

уравнение с модулем рисунок 36


Пример 2. Решить уравнение |2− 1| = 5− 10

Решение

Решим это уравнение таким же образом, как и предыдущее. Введём условие, требующее чтобы правая часть была больше либо равна нулю:

уравнение с модулем рисунок 18

В данном случае только значение 3 удовлетворяет условию ≥ 2. Оно же является единственным корнем исходного уравнения. Проверка показывает это:

уравнение с модулем рисунок 19

А число 11 na 7 не удовлетворяет условию ≥ 2 и не является корнем исходного уравнения. Проверка также показывает это:

уравнение с модулем рисунок 20

Видим, что модуль стал равен отрицательному числу, а это противоречит определению модуля и нашему условию ≥ 2.


Пример 3. Решить уравнение уравнение с модулем рисунок 34

Решение

Это уравнение мы решили, когда учились решать уравнения с модулем внутри которых другой модуль. Теперь данное уравнение можно решить, сведя его в совокупность.

Для начала перенесём x в правую часть, изменив знак:

уравнение с модулем рисунок 30

Теперь сведём данное уравнение в совокупность. Дополнительно введём условие в виде неравенства 6 − ≥ 0

уравнение с модулем рисунок 31

В левой части первого уравнения оставим модуль, остальные члены перенесём в правую часть. Тоже самое сделаем и со вторым уравнением. Также будем решать неравенство 6 − ≥ 0, оно позволит в конце проверять найденные корни на соответствие:

уравнение с модулем рисунок 32

Решим первое уравнение. Оно распадётся на следующую совокупность:

уравнение с модулем рисунок 33

Получились корни −2 и 8. Из них только −2 удовлетворяет условию ≤ 6.

Теперь решим второе уравнение. Оно является уравнением, содержащим переменную в правой части. При сведении его в совокупность дополним его условием −7 + 2≥ 0

уравнение с модулем рисунок 34

Дальнейшее элементарно:

уравнение с модулем рисунок 35

При решении второго уравнения получились корни 10 na 3 и 4. Прежде чем сверять их с условием  6 следует сверить их с условием уравнение с модулем рисунок 97 под которое решалось уравнение |3 − x| = −7 + 2x. Условию уравнение с модулем рисунок 97 удовлетворяет только корень 4.

В итоге корнями исходного уравнения уравнение с модулем рисунок 34 являются числа −2 и 4.


Пример 4. Решить уравнение |4x + 20| = −6x

Решение

На первый взгляд покажется, что данное уравнение не имеет решений, потому что правая часть отрицательна. Но это не совсем так. Правая часть содержит переменную x, которая может принять отрицательное значение или ноль, и это приведёт к тому что правая часть станет положительной либо равной нулю. А такое уравнение имеет право на существование.

В данном случае мы решим это уравнение, сведя его в совокупность. Но при этом укажем, что правая часть должна быть больше или равна нулю:

уравнение с модулем рисунок 39

Из найденных корней только корень −2 удовлетворяет исходному уравнению. Также он удовлетворяет нашему условию  0.

Ответ: −2.


Когда обе части — модули

Решим следующее уравнение:

|+ 7| = |1 + 3x|

Обе части этого уравнения являются модулями. Раскроем эти модули. Будем учитывать все возможные случаи при их раскрытии.

Случай 1. Если + 7 ≥ 0 и 1 + 3≥ 0, то модули в обеих частях раскроются со знаком плюс и тогда исходное уравнение примет вид:

x + 7 = 1 + 3x

Это простейшее линейное уравнение. Решим его:

уравнение с модулем рис 1

Случай 2. Если + 7 < 0 и 1 + 3< 0, то модули в обеих частях раскроются со знаком минус и тогда исходное уравнение примет вид:

−(x + 7) = −(1 + 3x)

Раскроем скобки, получим:

x − 7 = −1 − 3x

Замечаем, что если умножить обе части этого уравнения на −1, то получается уравнение x + 7 = 1 + 3x. А это уравнение мы получали в результате раскрытия модулей со знаком плюс.

То есть уравнения x + 7 = 1 + 3x и x − 7 = −1 − 3x являются равносильными, а значит имеют одни и те же корни. Убедимся в этом, решив уравнение  7 = −1  3x

уравнение с модулем рис 2

Поэтому, раскрыв модули со знаком плюс, нет необходимости раскрывать их со знаком минус, потому что в обоих случаях получаются уравнения, имеющие одни и те же корни.

Следующий случай это когда + 7 ≥ 0 и 1 + 3< 0. Тогда исходное уравнение примет вид + 7 = −1 − 3x. Найдём корень этого уравнения:

уравнение с модулем рис 4

И последний случай это когда + 7 < 0 и 1 + 3x ≥ 0. Тогда уравнение примет вид x − 7 = 1 + 3x. Если умножить это уравнение на −1, то получим уравнение + 7 = −1 − 3x. А это уравнение мы получали, когда рассматривали предыдущий случай (случай + 7 ≥ 0 и 1 + 3< 0).

Следовательно, уравнениеx − 7 = 1 + 3x равносильно предыдущему уравнению + 7 = −1 − 3x. Убедимся в этом решив уравнение −x − 7 = 1 + 3x

уравнение с модулем рис 5

Значит раскрыв левую часть со знаком плюс, а правую часть со знаком минус, нет необходимости раскрывать левую часть со знаком минус, а правую часть со знаком плюс, потому что в обоих случаях получаются уравнения, имеющие одни и те же корни.

Вообще, если в уравнении обе части являются модулями как в данном примере, то это уравнение можно свести в следующую совокупность:

уравнение с модулем рисунок 38

В этой конструкции уравнение вида |a| = |b| сведено в совокупность из двух уравнений = b и = b. Видно что первое уравнение получается путем раскрытия обоих модулей со знаком плюс, а второе уравнение — путем раскрытия модуля |a| со знаком плюс, а модуля|b|— со знаком минус.

Важно. Данная схема работает только тогда, когда обе части являются модулями без посторонних членов. Проще говоря, если будет дано уравнение, например |a| = |b| + c, то приведенную схему использовать нельзя.


Пример 2. Решить уравнение |2 − 3x| = |+ 5|

Решение

Обе части данного уравнения являются модулями. Воспользуемся схемой:

уравнение с модулем рисунок 38

У нас получится совокупность из двух уравнений. В первом уравнении оба модуля будут раскрыты со знаком плюс, во втором уравнении — модуль |2  3x| будет раскрыт со знаком плюс, а модуль |+ 5| со знаком минус:

уравнение с модулем рисунок 13

Выполним проверку:

уравнение с модулем рисунок 37

Ответ: Минус три четвертых  и  Семь вторых


Пример 3. Решить уравнение |x− 13+ 35|=|35 − x2

Решение

Обе части данного уравнения являются модулями. Воспользуемся схемой:

уравнение с модулем рисунок 38

У нас получится совокупность из двух уравнений. В первом уравнении оба модуля будут раскрыты со знаком плюс. Во втором уравнении — модуль |x− 13+ 35| будет раскрыт со знаком плюс, а модуль |35 − x2| со знаком минус:

уравнение с модулем рисунок 73

Приведём подобные члены в обоих уравнениях:

уравнение с модулем рисунок 74

Первое уравнение является неполным квадратным. Решим его, вынеся x за скобки. Второе уравнение решается элементарно:

уравнение с модулем рисунок 75

Ответ:   70 na 13,  , 0.


Когда решение — числовой промежуток

Нередко приходиться решать уравнения с модулем, где корнями являются не один или два числа, а числовой промежуток. Таковым, например, является уравнение:

|5+ 3| = −5− 3

Раскроем модуль этого уравнения:

уравнение с модулем рисунок 47

Если раскрыть модуль со знаком плюс, то получается уравнение 5+ 3 = −5 3. Решим его:

уравнение с модулем рисунок 48

А если раскрыть модуль со знаком минус, то получится уравнение −5 3 = −5 3. В этом уравнении обе части являются одинаковыми, а значит данное равенство является тождеством. Оно будет верно при любом значении x. Значит корнями уравнения −5 3 = −5 3 являются все числа от минус бесконечности до плюс бесконечности:

x ∈ (−∞; +∞)

Но надо помнить про условия, согласно которым были раскрыты модули. В первом случае мы получили корень Минус три пятых. Он будет верен только при условии что уравнение с модулем рисунок 62. Это условие соблюдено. Проверка также показывает что корень подходит:

уравнение с модулем рисунок 61

Значит один из корней уравнений равен Минус три пятых

Во втором случае мы получили множество корней от минус бесконечности до плюс бесконечности. Но это будет верно только при условии что уравнение с модулем рисунок 63

Например, если взять любое число из промежутка (−∞; +∞), но которое не будет удовлетворять условию уравнение с модулем рисунок 63, то это число не будет обращать наше уравнение в верное равенство.

Например, число 2 принадлежит промежутку (−∞; +∞), но не удовлетворяет условию уравнение с модулем рисунок 63, а значит число 2 не является корнем исходного уравнения. Проверка также покажет это:

уравнение с модулем рисунок 64

А если взять к примеру число −5, то оно будет принадлежать промежутку (−∞; +∞) и удовлетворять условию уравнение с модулем рисунок 63, а значит будет обращать исходное уравнение в верное равенство:

уравнение с модулем рисунок 65

Поэтому ответ надо записать так, чтобы были выполнены оба условия уравнение с модулем рисунок 62  и уравнение с модулем рисунок 63. Для наглядности нарисуем координатную прямую и обозначим её как x

уравнение с модулем рисунок 49Отметим на ней наш первый корень Минус три пятых

уравнение с модулем рисунок 66

Раскрыв модуль со знаком минус и решив получившееся уравнение, мы получили в ответе множество всех чисел от минус бесконечности до плюс бесконечности, но при этом было дано условие уравнение с модулем рисунок 63. Значит более точным ответ в этом случае будет таким:

Корнями уравнения −5x − 3 = −5x − 3 при условии уравнение с модулем рисунок 63 являются все числа от минус бесконечности до Минус три пятых

Значит на координатной прямой нужно заштриховать область слева от числа Минус три пятых. Они будут иллюстрировать числа, меньшие Минус три пятых

уравнение с модулем рисунок 67

Число Минус три пятых тоже является верным корнем исходного уравнения. Он был получен при раскрытии модуля со знаком плюс. Поэтому на координатной прямой пустой кружок нужно закрасить. Так мы включим число Минус три пятых во множество решений:

Тогда окончательный ответ будет выглядеть так:

уравнение с модулем рисунок 69

Ответ: уравнение с модулем рисунок 69

Также, можно решить это уравнение сведя его в совокупность, дополнительно указав, что правая часть должна быть больше либо равна нулю:

уравнение с модулем рисунок 119


Пример 2. Решить уравнение |2 3| = 3  2x

Решение

уравнение с модулем рисунок 120

Решим исходное уравнение для случаев когда 2 3 ≥ 0 и 2 3 < 0

уравнение с модулем рисунок 121

уравнение с модулем рисунок 122

Ответ: уравнение с модулем рисунок 123


Использование координатной прямой

Рассмотрим ещё один способ решения элементарных уравнений с модулем — с помощью координатной прямой. Этот способ используется редко, но знать о нём не помешает.

Решим наше самое первое уравнение |− 2| = 5 с помощью координатной прямой. Напомним, что корнями этого уравнения были числа 7 и −3.

Модуль есть расстояние от начала координат до точки A. Либо расстояние между двумя числами на координатной прямой.

Расстояние между двумя числами выражается в виде разности |x1x2|, где x1 — первое число, x2 — второе число.

Если внимательно посмотреть на уравнение |x − 2|= 5, то можно увидеть что его левая часть это расстояние от x до 2 (или от 2 до x) и это расстояние равно 5. Отмéтим на координатной прямой число x и число 2

модуль числа рисунок 25

Правая часть уравнения |x − 2|= 5 говорит о том, что расстояние от x до 2 составляет пять единиц:

модуль числа рисунок 26

Если расстояние от x до 2 равно 5, то и расстояние от 2 до x тоже равно 5. Это позволяет отсчитать пять целых шагов от числа 2 к числу x и таким образом узнать значение x

модуль числа рисунок 34

Видно, что отсчитав пять шагов влево мы попали в точку с координатой −3. А это один из корней, который мы находили для уравнения |x − 2|= 5.

Но пять целых шагов от числа 2 можно отсчитать не только влево, но и вправо:

модуль числа рисунок 35

Если отсчитать пять целых шагов вправо, то попадём в точку с координатой 7. Это тоже был корень уравнения |− 2|= 5

модуль числа рисунок 23


Несколько модулей в одной части

Решим следующее уравнение:

| 5|  |x| = 1

Это уравнение содержит два модуля в левой части. Чтобы решить данное уравнение нужно раскрыть его модули. Рассмотреть нужно каждый из случаев:

  • когда оба модуля больше либо равны нулю;
  • когда оба модуля меньше нуля;
  • когда первый модуль больше либо равен нулю, а второй модуль меньше нуля;
  • когда первый модуль меньше нуля, а второй модуль больше либо равен нулю.

Не будем комментировать каждый случай, а сразу приведём решение:

uravnenie-s-modulem-risunok-52

Первые два случая корней не дали. В третьем случае нашелся корень 3, но он не удовлетворяет условиям  5  0 и < 0, поэтому не является корнем исходного уравнения.

В четвёртом случае нашёлся корень 2, который удовлетворяет условиям  5 < 0 и ≥ 0. Также он удовлетворяет исходному уравнению.

Заметно, что такой способ решения уравнения неудобен. Если модулей в уравнении будет три, четыре или более, то придётся рассматривать намного больше случаев. Человек запутавшись, может забыть рассмотреть какой-то из случаев, и получится что уравнение решено не полностью.

Поэтому такой вид уравнения как в данном примере удобнее решать методом интервалов. Об этом мы поговорим в следующем уроке.

Задания для самостоятельного решения

Задание 1. Решить уравнение:
Решение:
Задание 2. Решить уравнение:
Решение:
Задание 3. Решить уравнение:
Решение:
Задание 4. Решить уравнение:
Решение:
Ответ: .
Задание 5. Решить уравнение:
Решение:
Ответ: .
Задание 6. Решить уравнение:
Решение:
Ответ: .
Задание 7. Решить уравнение:
Решение:
Ответ: .
Задание 8. Решить уравнение:
Решение:
Ответ: .
Задание 9. Решить уравнение:
Решение:
Ответ: .
Задание 10. Решить уравнение:
Решение:
Ответ: .
Задание 11. Решить уравнение:
Решение:
Ответ: .
Задание 12. Решить уравнение:
Решение:
Ответ: 0, 5.

Понравился урок?
Вступай в нашу новую группу Вконтакте и начни получать уведомления о новых уроках

Возникло желание поддержать проект?
Используй кнопку ниже

Обобщённое понятие модуля числа

В данном уроке мы рассмотрим понятие модуля числа более подробно.

Что такое модуль?

Модуль — это расстояние от начала координат до какого-нибудь числа на координатной прямой. Поскольку расстояние не бывает отрицательным, то и модуль всегда неотрицателен. Так, модуль числа 3 равен 3, как и модуль числа −3 равен 3

| 3 |= 3

|−3|= 3

Предстáвим, что на координатной прямой расстояние между целыми числами равно одному шагу. Теперь если отметить числа −3 и 3, то расстояние до них от начала координат будет одинаково равно трём шагам:

модуль числа рисунок 10

Модуль это не только расстояние от начала координат до какого-нибудь числа. Модуль это также расстояние между любыми двумя числами на координатной прямой. Такое расстояние выражается в виде разности между этими числами, заключенной под знак модуля:

|x1x2|

Где x1 и x2 — числа на координатной прямой.

Например, отметим на координатной прямой числа 2 и 5.

модуль числа рисунок 26

Расстояние между числами 2 и 5 можно записать с помощью модуля. Для этого запишем разность из чисел 2 и 5 и заключим эту разность под знак модуля:

|2 − 5| = |−3| = 3

Видим, что расстояние от числа 2 до числа 5 равно трём шагам:

модуль числа рисунок 27

Если расстояние от 2 до 5 равно 3, то и расстояние от 5 до 2 тоже равно 3

модуль числа рисунок 28

То есть, если в выражении |5 − 2| поменять числа местами, то результат не изменится:

|5 − 2| = | 3 | = 3

Тогда можно записать, что |2 − 5| = |5 − 2|. Вообще, справедливо следующее равенство:

|x1 x2| = |x2 x1|

Это равенство можно прочитать так: Расстояние от x1 до x2 равно расстоянию от x2 до x1.


Раскрытие модуля

Когда мы говорим, что |3|= 3 или |−3|= 3 мы выполняем действие называемое раскрытием модуля.

Правило раскрытия модуля выглядит так:

модуль числа свойство

Такую запись мы ранее не использовали. Дело в том, что равенство можно задавать несколькими формулами. Фигурная скобка указывает, что возможны два случая в зависимости от условия. В данном случае условиями являются записи «если x ≥ и «если x < .

В зависимости от того что будет подставлено вместо x, выражение |x| будет равно x, если подставленное число больше или равно нулю. А если вместо x подставлено число меньшее нуля, то выражение |x| будет равно −x.

Второй случай на первый взгляд может показаться противоречивым, поскольку запись |x| = −x выглядит будто модуль стал равен отрицательному числу. Следует иметь ввиду, что когда x < 0, то под знáком модуля располагается отрицательное число. После знака равенства нужно подстáвить данное отрицательное число вместо x и раскрыть скобки.

Например, найдём модуль числа −7, используя правило раскрытия модуля:

модуль числа свойство

Итак, = −7

|−7|

В данном случае выполняется второе условие < 0, ведь −7 < 0

модуль числа рисунок 5

Поэтому используем вторую формулу. А именно |x| = −x. Подстáвим вместо x число −7

модуль числа рисунок 7

Отсюда:

модуль числа рисунок 6

Поэтому |−7| = 7.


Пример 2. Пусть = 5. То есть мы рассматриваем модуль числа 5

| 5 |

В данном случае выполняется первое условие ≥ 0, ведь 5 ≥ 0

модуль числа рисунок 8

Поэтому используем первую формулу. А именно | x | = x. Получаем | 5 | = 5.

Ноль это своего рода точка перехода, в которой модуль меняет свой порядок раскрытия и далее сохраняет свой знак. Визуально это можно представить так:

модуль рисунок 1

На рисунке красные знаки минуса и плюса указывают как будет раскрываться модуль |x| на промежутках < 0 и ≥ 0.

К примеру, если взять числа 1, 9 и 13, а они принадлежат промежутку ≥ 0, то согласно рисунку модуль  |x| раскроется со знаком плюс:

| 1 | = 1

| 9 | = 9

| 13 | = 13

А если возьмём числа, меньшие нуля, например −3, −9, −15, то согласно рисунку модуль раскроется со знаком минус:

|−3| = −(−3) = 3

|−9| = −(−9) = 9

|−15| = −(−15) = 15


Пример 3. Пусть = √4 − 6. То есть мы рассматриваем модуль выражения √4 − 6,

|√4 − 6|

Корень из числа 4 равен 2. Тогда модуль примет вид

|√4 − 6| = |2 − 6| = |−4|

x который был равен √4−6 теперь стал равен −4. В данном случае выполняется второе условие < 0, ведь −4 < 0

модуль числа рисунок 5

Следовательно, используем вторую формулу |x| = −x. Продолжаем решение в исходном примере:

|√4 − 6| = |2 − 6| = |−4| = −(−4) = 4

На практике обычно рассуждают так:

«Модуль раскрывается со знаком плюс, если подмодульное выражение больше или равно нулю; модуль раскрывается со знаком минус, если подмодульное выражение меньше нуля».

Примеры:

|2| = 2 — модуль раскрылся со знаком плюс, поскольку 2 ≥ 0

|−4| = −(−4) = 4 — модуль раскрылся со знаком минус, поскольку −4 < 0

В некоторых учебниках можно встретить следующую запись правила раскрытия модуля:

модуль числа рисунок 16

В этой записи первое условие которое ранее выглядело как ≥ 0 расписано подробнее, а именно сказано что если > 0, то выражение |x| будет равно x, а если x=0, то выражение |x| будет равно нулю.

Пример 4. Пусть x = 0. То есть мы рассматриваем модуль нуля:

| 0 |

В данном случае выполняется условие x=0, ведь 0 = 0

 

модуль числа рисунок 17

Отсюда: |0| = 0


Пример 5. Раскрыть модуль в выражении |x|+ 3

Если ≥ 0, то модуль раскроется со знаком плюс, и тогда исходное выражение примет вид + 3.

Если < 0, то модуль раскроется со знаком минус, и тогда исходное выражение примет вид −+ 3. Чтобы сделать это выражение более удобным для восприятия, поменяем местами его члены, полýчим 3 − x

Теперь запишем решение так:

модуль числа рисунок 47

Проверим это решение при произвольных значениях x.

Допустим, требуется найти значение выражения |x|+ 3 при = 5. Поскольку 5 ≥ 0, то модуль, содержащийся в выражении |x|+ 3 раскрóется со знаком плюс и тогда решение примет вид:

|x|+ 3 = x + 3 = 5 + 3 = 8

Найдём значение выражения |x|+ 3 при = −6. Поскольку −6 < 0, то модуль содержащийся в выражении |x|+ 3 раскроется со знаком минус и тогда решение примет вид:

|x| + 3 = 3 − x = 3 − (−6) = 9


Пример 6. Раскрыть модуль в выражении x +|x + 3|

Если x + 3 ≥ 0, то модуль |x + 3| раскроется со знаком плюс и тогда исходное выражение примет вид x + x + 3, откуда 2x + 3.

Если x + 3 < 0, то модуль |x + 3| раскроется со знаком минус и тогда исходное выражение примет вид x − (x + 3), откуда x − x − 3 = −3.

Запишем решение так:

модуль числа рисунок 5

Заметим, что условия x + 3 ≥ 0 и x + 3 < 0 являются неравенствами. Их можно привести к более простому виду, решив их:

модуль числа рисунок 6

Тогда условия из решения можно заменить на равносильные x ≥ −3 и x < −3

модуль числа рисунок 7

Во втором случае когда x строго меньше −3 выражение x +|x + 3| всегда будет равно постоянному числу −3.

Например, найдём значение выражения x +|x + 3| при x = −5. Поскольку −5 < −3, то согласно нашему решению значение выражения x +|x + 3| будет равно −3

При x = −5,
x +|x + 3| = x − x − 3 = −5(−5) 3 = −3

Найдём значение выражения x +|x + 3| при = 4. Поскольку 4 ≥ −3, то согласно нашему решению модуль выражения+|+ 3| раскрывается со знаком плюс, и тогда исходное выражение принимает вид 2x+3, откуда подставив 4 получим 11

При x = 4,
x +|x + 3| = 2x+3 = 2 × 4 + 3 = 8 + 3 = 11

Найдём значение выражения x +|x + 3| при x=−3.

Поскольку −3 ≥ −3, то согласно нашему решению модуль выражения+|+ 3| раскрывается со знаком плюс, и тогда исходное выражение принимает вид 2x+3, откуда подставив −3 получим −3

x +|x + 3| = 2+ 3 = 2 × (−3) + 3 = −6 + 3 = −3


Пример 3. Раскрыть модуль в выражении модуль числа рисунок 8

Как и прежде используем правило раскрытия модуля:

модуль числа рисунок 10

Но это решение не будет правильным, поскольку в первом случае написано условие ≥ 0, которое допускает что при x = 0 знаменатель выражения модуль числа рисунок 8 обращается в ноль, а на ноль делить нельзя.

В данном примере удобнее использовать подробную запись правила раскрытия модуля, где отдельно рассматривается случай при котором = 0

модуль числа рисунок 16

Перепишем решение так:

модуль числа рисунок 9

В первом случае написано условие > 0. Тогда выражение модуль числа рисунок 8 станет равно 1. Например, если = 3, то числитель и знаменатель станут равны 3, откуда полýчится 1

модуль числа рисунок 11

И так будет при любом x, бóльшем нуля.

Во втором случае написано условие x = 0. Тогда решений не будет, потому что на ноль делить нельзя.

В третьем случае написано условие x < 0. Тогда выражение модуль числа рисунок 8 станет равно −1. Например, если = −4, то числитель станет равен 4, а знаменатель −4, откуда полýчится единица −1

модуль числа рисунок 12


Пример 4. Раскрыть модуль в выражении модуль числа рисунок 18

Если ≥ 0, то модуль, содержащийся в числителе, раскроется со знаком плюс, и тогда исходное выражение примет вид модуль числа рисунок 19, которое при любом x, бóльшем нуля, будет равно единице:

модуль числа рисунок 20

Если < 0, то модуль раскроется со знаком минус, и тогда исходное выражение примет вид модуль числа рисунок 21

модуль числа рисунок 22

Но надо учитывать, что при = − 1 знаменатель выражения модуль числа рисунок 18 обращается в ноль. Поэтому второе условие < 0 следует дополнить записью о том, какие значения может принимать x

модуль числа рисунок 23

Преобразование выражений с модулями

Модуль, входящий в выражение, можно рассматривать как полноценный множитель. Его можно сокращать и выносить за скобки. Если модуль входит в многочлен, то его можно сложить с подобным ему модулем.

Как и у обычного буквенного множителя, у модуля есть свой коэффициент. Например, коэффициентом модуля |x| является 1, а коэффициентом модуля −|x| является −1. Коэффициентом модуля 3|x+1| является 3, а коэффициентом модуля −3|x+1| является −3.

Пример 1. Упростить выражение |x| + 2|x| − 2+ 5y и раскрыть модуль в получившемся выражении.

Решение

Выражения|x| и 2|x| являются подобными членами. Слóжим их. Остальное оставим без изменений:

модуль числа рисунок 48

Раскроем модуль в получившемся выражении. Если x ≥ 0, то получим 3− 2+ 5y, откуда + 5y.

Если < 0, то получим −3x − 2+ 5y, откуда −5+ 5y. Вынесем за скобки множитель −5, получим −5(x − y)

В итоге имеем следующее решение:

модуль числа рисунок 24


Пример 2. Раскрыть модуль в выражении: −|x|

Решение

В данном случае перед знаком модуля стоит минус. Его можно понимать как минус единицу перед знаком модуля. Если x ≥ 0, то модуль раскроется со знаком плюс, и тогда исходное выражение примет вид x

Если < 0, то модуль раскроется со знаком минус, и тогда исходное выражение примет вид −(−x) откуда получим просто x

Задания для самостоятельного решения

Задание 1. Раскройте модуль:
Решение:
Задание 2. Раскройте модуль:
Решение:
Задание 3. Раскройте модуль:
Решение:
Задание 4. Раскройте модуль:
Решение:
Задание 5. Раскройте модуль:
Решение:

Понравился урок?
Вступай в нашу новую группу Вконтакте и начни получать уведомления о новых уроках

Возникло желание поддержать проект?
Используй кнопку ниже

Разложение квадратного трёхчлена на множители

Как разложить на множители квадратный трёхчлен

Квадратный трёхчлен — это многочлен вида axbx c.

В прошлых уроках мы решали квадратные уравнения. Общий вид таких уравнений выглядел так:

axbx c = 0

Левая часть этого уравнения является квадратным трёхчленом.

Одним из полезных преобразований при решении задач является разложение квадратного трёхчлена на множители. Для этого исходный квадратный трёхчлен приравнивают к нулю и решают квадратное уравнение. В этом случае говорят, что выполняется поиск корней квадратного трёхчлена.

Полученные корни x1 и x2 следует подстáвить в следующее выражение, которое и станет разложением:

a(− x1)(− x2)

Таким образом, чтобы разложить квадратный трёхчлен на множители при помощи решения квадратного уравнения, нужно воспользоваться следующей готовой формулой:

axbx c = a(− x1)(− x2)

Где левая часть — исходный квадратный трёхчлен.

Пример 1. Разложить на множители следующий квадратный трёхчлен:

x− 8+ 12

Найдём корни квадратного трёхчлена. Для этого приравняем данный квадратный трёхчлен к нулю и решим квадратное уравнение:

x− 8+ 12 = 0

В данном случае коэффициент b является чётным. Поэтому можно воспользоваться формулами для чётного второго коэффициента. Чтобы сэкономить время, некоторые подробные вычисления можно пропустить:

разложение квадратного трехчлена на множители рис 2

Итак, x= 6, x= 2. Теперь воспользуемся формулой:

axbx c = a(− x1)(− x2)

В левой части вместо выражения axbx c напишем свой квадратный трёхчлен x− 8x + 12. А в правой части подставим имеющиеся у нас значения. В данном случае = 1, x= 6, x= 2

x− 8x + 12 = 1(x − 6)(x − 2) = (x − 6)(x − 2)

Если a равно единице (как в данном примере), то решение можно записать покороче:

x− 8x + 12 = (x − 6)(x − 2)

Чтобы проверить правильно ли разложен квадратный трёхчлен на множители, нужно раскрыть скобки у правой части получившегося равенства.

Раскроем скобки у правой части равенства, то есть в выражении (x − 6)(x − 2). Если мы всё сделали правильно, то должен получиться квадратный трёхчлен x− 8x + 12

(x − 6)(x − 2) = x− 6− 2+ 12 = x− 8+ 12


Пример 2. Разложить на множители следующий квадратный трёхчлен:

2x− 14+ 24

Приравняем данный квадратный трёхчлен к нулю и решим уравнение:

2x− 14+ 24 = 0

Как и в прошлом примере коэффициент b является чётным. Поэтому можно воспользоваться формулами для чётного второго коэффициента:

разложение квадратного трехчлена на множители рис 1

Итак, x= 4, x= 3. Приравняем квадратный трехчлен 2x− 14+ 24 к выражению a(− x1)(− x2), где вместо переменных a, x1 и x2 подстáвим соответствующие значения. В данном случае = 2

2x− 14+ 24 = 2(− 4)(− 3)

Выполним проверку. Для этого раскроем скобки у правой части получившегося равенства. Если мы всё сделали правильно, то должен получиться квадратный трёхчлен 2x− 14+ 24

2(− 4)(− 3) = 2(x− 4−3+ 12) = 2(x− 7+ 12) = 2x− 14+ 24


Как это работает

Разложение квадратного трёхчлена на множители происходит, если вместо коэффициентов квадратного трёхчлена подстáвить теорему Виета и выполнить тождественные преобразования.

Для начала рассмотрим случай, когда коэффициент a квадратного трёхчлена равен единице:

xbx c

Вспоминаем, что если квадратное уравнение является приведённым, то теорема Виета имеет вид:

Тогда приведённый квадратный трехчлен xbx c можно разложить на множители следующим образом. Сначала выразим b из уравнения xx= −b. Для этого можно умножить обе его части на −1

разложение квадратного трехчлена на множители рис 12

Переменную c из теоремы Виета выражать не нужно — она уже выражена. Достаточно поменять местами левую и правую часть:

разложение квадратного трехчлена на множители рис 13

Теперь подставим выраженные переменные b и c в квадратный трёхчлен xbx c

разложение квадратного трехчлена на множители рис 14

Раскроем скобки там где это можно:

разложение квадратного трехчлена на множители рис 15

В получившемся выражении выполним разложение многочлена на множители способом группировки. В данном случае удобно сгруппировать первый член со вторым, а третий с четвёртым:

разложение квадратного трехчлена на множители рис 16

Из первых скобок вынесем общий множитель x, из вторых скобок — общий множитель x2

разложение квадратного трехчлена на множители рис 17

Далее замечаем, что выражение (− x1) является общим множителем. Вынесем его за скобки:

разложение квадратного трехчлена на множители рис 18

Мы пришли к тому, что выражение xbx c стало равно (− x1)(− x2)

xbx c(− x1)(− x2)

Но это был случай, когда исходный квадратный трёхчлен является приведённым. В нём коэффициент a равен единице. И соответственно, в формуле разложения такого квадратного трехчлена коэффициент a можно опустить.

Теперь рассмотрим случай, когда коэффициент a квадратного трёхчлена не равен единице. Это как раз тот случай, когда в формуле разложения присутствует перед скобками коэффициент a

axbx c = a(− x1)(− x2)

Вспоминаем, что если квадратное уравнение не является приведённым, то есть имеет вид axbx = 0, то теорема Виета принимает следующий вид:

Теорема Виета рисунок 66

Это потому что теорема Виета работает только для приведённых квадратных уравнений. А чтобы уравнение axbx = 0 стало приведённым, нужно разделить обе его части на a

квадратное уравнение рисунок 122

Далее чтобы квадратный трёхчлен вида axbx c разложить на множители, нужно вместо b и c подставить соответствующие выражения из теоремы Виета. Но в этот раз нам следует использовать равенства разложение квадратного трехчлена на множители рис 19 и разложение квадратного трехчлена на множители рис 20

Для начала выразим b и c. В первом равенстве умножим обе части на a. Затем обе части получившегося равенства умножим на −1

разложение квадратного трехчлена на множители рис 3

Теперь из второго равенства выразим c. Для этого умножим обе его части на a

разложение квадратного трехчлена на множители рис 4

Теперь подставим выраженные переменные b и с в квадратный трёхчлен axbx c. Для наглядности каждое преобразование будем выполнять на новой строчке:

разложение квадратного трехчлена на множители рис 5

Здесь вместо переменных b и c были подставлены выражения −ax− ax2 и ax1x2, которые мы ранее выразили из теоремы Виета. Теперь раскроем скобки там где это можно:

разложение квадратного трехчлена на множители рис 6

В получившемся выражении выполним разложение многочлена на множители способом группировки. В данном случае удобно сгруппировать первый член со вторым, а третий с четвёртым:

разложение квадратного трехчлена на множители рис 7

Теперь из первых скобок вынесем общий множитель ax, а из вторых — общий множитель −ax2

разложение квадратного трехчлена на множители рис 9

Далее замечаем, что выражение x − x1 тоже является общим множителем. Вынесем его за скобки:

разложение квадратного трехчлена на множители рис 10

Вторые скобки содержат общий множитель a. Вынесем его за скобки. Его можно расположить в самом начале выражения:

разложение квадратного трехчлена на множители рис 11

Мы пришли к тому, что выражение axbx c стало равно a(− x1)(− x2)

axbx c = a(− x1)(− x2)

Отметим, что если квадратный трехчлен не имеет корней, то его нельзя разложить на множители. Действительно, если не найдены корни квадратного трёхчлена, то нéчего будет подставлять в выражение a(− x1)(− x2) вместо переменных x1 и x2.


Примеры разложений

Пример 1. Разложить на множители следующий квадратный трёхчлен:

3x− 2− 1

Найдём корни квадратного трёхчлена:

разложение квадратного трехчлена на множители рис 21

Воспользуемся формулой разложения. В левой части напишем квадратный трёхчлен 3x− 2− 1, а в правой части — его разложение в виде a(− x1)(− x2), где вместо a, x1 и x2 подстáвим соответствующие значения:

разложение квадратного трехчлена на множители рис 23

разложение квадратного трехчлена на множители рис 22

Во вторых скобках можно заменить вычитание сложением:

разложение квадратного трехчлена на множители рис 24


Пример 2. Разложить на множители следующий квадратный трёхчлен:

− 11x + 6x2

Упорядочим члены так, чтобы старший коэффициент располагался первым, средний — вторым, свободный член — третьим:

6x2 − 11x + 3

Найдём корни квадратного трёхчлена:

разложение квадратного трехчлена на множители рис 25

Воспользуемся формулой разложения:

разложение квадратного трехчлена на множители рис 26

Упростим получившееся разложение. Вынесем за первые скобки общий множитель 3

разложение квадратного трехчлена на множители рис 27

Теперь воспользуемся сочетательным законом умножения. Напомним, что он позволяет перемножать сомножители в любом порядке. Умножим 3 на вторые скобки. Это позвóлит избавиться от дроби в этих скобках:

разложение квадратного трехчлена на множители рис 28


Пример 3. Разложить на множители следующий квадратный трёхчлен:

3x7x − 6

Найдём корни квадратного трёхчлена:

разложение квадратного трехчлена на множители рис 29

Воспользуемся формулой разложения:

разложение квадратного трехчлена на множители рис 30


Пример 4. Найдите значение k, при котором разложение на множители трёхчлена 3x− 8k содержит множитель (− 2)

Если разложение содержит множитель (− 2), то один из корней квадратного трёхчлена равен 2. Пусть корень 2 это значение переменной x1

разложение квадратного трехчлена на множители рис 31

Чтобы найти значение k, нужно знать чему равен второй корень. Для его определения воспользуемся теоремой Виета.

В данном случае квадратный трёхчлен не является приведённым, поэтому сумма его корней будет равна дроби 8 na 3, а произведение корней — дроби k na 3

разложение квадратного трехчлена на множители рис 32

Выразим из первого равенства переменную x2 и сразу подстáвим найденное значение во второе равенство вместо x2

разложение квадратного трехчлена на множители рис 33

Теперь из второго равенства выразим k. Так мы найдём его значение.

разложение квадратного трехчлена на множители рис 34


Пример 5. Разложить на множители следующий квадратный трёхчлен:

разложение квадратного трехчлена на множители рис 35

Перепишем данный трёхчлен в удобный для нас вид. Если в первом члене заменить деление умножением, то получим разложение квадратного трехчлена на множители рис 36. Если поменять местами сомножители, то получится 1 na 2 x v 2. То есть коэффициент a станет равным одна вторая

Коэффициент b можно перевести в обыкновенную дробь. Так проще будет искать дискриминант:

разложение квадратного трехчлена на множители рис 37

Найдём корни квадратного трёхчлена:

разложение квадратного трехчлена на множители рис 38

Воспользуемся формулой разложения:


Когда корень только один

Если квадратный трёхчлен имеет только один корень, то формула разложения квадратного трёхчлена примет такой вид:

axbx c = a(− x1)2

Вместо x1 подставляется единственный найденный корень квадратного трёхчлена.

Например, квадратный трёхчлен x− 6+ 9 имеет только один корень. Дискриминант этого квадратного трёхчлена равен нулю:

D = b− 4ac = (−6)− 4 × 1 × 9 = 36 − 36 = 0

и далее вычисляется единственный корень по известной формуле:

Тогда по формуле, которая приведена выше, получим:

axbx c = a(− x1)2

x− 6+ 9 = (x − 3)2

Внимательные наверное сразу поняли почему происходит именно так. Если дискриминант квадратного трёхчлена равен нулю, то корень вычисляется по формуле:

квадратное уравнение рисунок 95

Но никто не запрещает нам использовать формулы вычисления корней квадратного трёхчлена:

формула для вычисления первого корня квадратного уравнения

формула для вычисления второго корня квадратного уравнения

Просто в этом случае мы получим один и тот же корень 3

разложение квадратного трехчлена на множители рис 1

Видим, что xx2. Теперь для квадратного трёхчлена x− 6+ 9 применим нашу формулу разложения, которую мы применяем когда два корня:

x− 6+ 9 = (x − 3)(x − 3)

Выражение (x − 3)(x − 3) это перемножение двух сомножителей, каждый из которых равен (x − 3). Это выражение можно заменить на выражение (x − 3)2

x− 6+ 9 = (x − 3)2


Задания для самостоятельного решения

Задание 1. Разложить на множители квадратный трёхчлен:
Решение:
Задание 2. Разложить на множители квадратный трёхчлен:
Решение:
Задание 3. Разложить на множители квадратный трёхчлен:
Решение:
Задание 4. Разложить на множители квадратный трёхчлен:
Решение:
Задание 5. Разложить на множители квадратный трёхчлен:
Решение:
Задание 6. Разложить на множители квадратный трёхчлен:
Решение:
Задание 7. Разложить на множители квадратный трёхчлен:
Решение:
Задание 8. Разложить на множители квадратный трёхчлен:
Решение:
Задание 9. Разложить на множители квадратный трёхчлен:
Решение:
Задание 10. Разложить на множители квадратный трёхчлен:
Решение:
Задание 11. Разложить на множители квадратный трёхчлен:
Решение:
Задание 12. Разложить на множители квадратный трёхчлен:
Решение:

Понравился урок?
Вступай в нашу новую группу Вконтакте и начни получать уведомления о новых уроках

Возникло желание поддержать проект?
Используй кнопку ниже

Теорема Виета

Что называют теоремой?

Если человек обнаружил в математике какую-нибудь закономерность, позволяющую быстро решить ту или иную задачу, то ему не следует говорить о том, что он сделал открытие. Потому что может случиться так, что эта закономерность работает только для определённых случаев, а для других не работает или вовсе решает задачу неправильно.

Чтобы поделиться своим открытием с другими людьми, найденную закономерность следует сформулировать в виде утверждения, а затем доказать это утверждение, приводя неоспоримые факты.

Сформулированное утверждение называют теоремой. А доказательство теоремы состоит из фактов, логических рассуждений и вычислений, которые не оспариваются.

Например, теоремой можно назвать следующее утверждение:

«Если числитель и знаменатель обыкновенной дроби умнóжить на какое-нибудь число, то значение данной дроби не измéнится».

А затем привести такое доказательство:

Пусть, имеется дробь a na b. Умнóжим числитель и знаменатель этой дроби на число с. Тогда полýчится дробь ac na bc. Докáжем, что дроби a na b и ac na bc равны. То есть докажем, что равенство a na b ravno ac na bc является верным.

Для доказательства этого равенства воспользуемся основным свойством пропорции:

Основное свойство дроби рисунок 1

От перестановки мест сомножителей произведение не меняется. Поэтому в получившемся равенстве можно упорядочить правую часть по алфавиту:

Основное свойство дроби рисунок 2

Поскольку равенство a na b ravno ac na bc является пропорцией, а пропорция это равенство двух отношений, то дроби a na b и ac na bc равны. Теорема доказана.


Теорема Виета

Французский математик Франсуа Виет выявил интересную взаимосвязь между коэффициентами приведённого квадратного уравнения и корнями этого же уравнения. Эта взаимосвязь представлена в виде теоремы и формулируется так:

Сумма корней приведённого квадратного уравнения xbx = 0 равна второму коэффициенту, взятому с противоположным знáком, а произведение корней равно свободному члену.

То есть, если имеется приведённое квадратное уравнение xbx = 0, а его корнями являются числа x1 и x2, то справедливы следующие два равенства:

Теорема Виета рисунок 23

Знак системы (фигурная скобка) говорит о том, что значения x1 и x2 удовлетворяют обоим равенствам.

Покажем теорему Виета на примере приведённого квадратного уравнения x+ 4+ 3 = 0.

Мы пока не знаем какие корни имеет уравнение x+ 4+ 3 = 0. Но по теореме Виета можно записать, что сумма этих корней равна второму коэффициенту 4, взятому с противоположным знáком. Если коэффициент 4 взять с противоположным знáком, то получим −4. Тогда:

Теорема Виета рисунок 55

А произведение корней по теореме Виета будет равно свободному члену. В уравнении x+ 4+ 3 = 0 свободным членом является 3. Тогда:

Теорема Виета рисунок 54

Теперь проверим действительно ли сумма корней равна −4, и равно ли произведение 3. Для этого найдём корни уравнения x+ 4+ 3 = 0. А для удобства воспользуемся формулами для чётного второго коэффициента:

Теорема Виета рисунок 58

Корнями уравнения являются числа −1 и −3. По теореме Виета их сумма должна была равняться второму коэффициенту уравнения x+ 4+ 3 = 0, взятому с противоположным знаком. Действительно, так оно и есть. Вторым коэффициентов в уравнении x+ 4+ 3 = 0 является 4. Если взять его с противоположным знаком и приравнять сумму корней xx2 к этому коэффициенту, то получается верное равенство:

Теорема Виета рисунок 59

А произведение корней −1 и −3 по теореме Виета должно было равняться свободному члену уравнения x+ 4+ 3 = 0, то есть числу 3. Видим, что это условие тоже выполняется:

Теорема Виета рисунок 60

Значит выражение Теорема Виета рисунок 54 является справедливым.


Рассмотрим квадратное уравнение x− 8+ 15 = 0. По теореме Виета сумма корней этого уравнения равна второму коэффициенту, взятому с противоположным знаком. Второй коэффициент равен −8. Если взять его с противоположным знаком, то получим 8. Тогда:

Теорема Виета рисунок 57

А произведение корней равно свободному члену. В уравнении x− 8+ 15 = 0 свободным членом является 15. Тогда:

Теорема Виета рисунок 56

Теперь проверим действительно ли сумма корней равна 8, и равно ли произведение 15. Для этого найдём корни данного уравнения. А для удобства воспользуемся формулами для чётного второго коэффициента. В этот раз пропустим нéкоторые подробные записи:

Теорема Виета рисунок 61

Видим, что корнями уравнения x− 8+ 15 = 0 являются числа 5 и 3. Их сумма равна 8. То есть сумма корней равна второму коэффициенту уравнения x− 8+ 15 = 0, взятому с противоположным знаком.

А произведение чисел 5 и 3 равно 15. То есть равно свободному члену уравнения x− 8+ 15 = 0.

Значит выражение Теорема Виета рисунок 56 является справедливым.

Замечание. Чтобы теорема Виета выполнялась, квадратное уравнение обязательно должно быть приведённым и иметь корни.

Например, рассмотрим квадратное уравнение x− 2+ 4 = 0. Напишем сумму и произведение корней этого уравнения:

Теорема Виета рисунок 61

Но уравнение x− 2+ 4 = 0 не имеет корней, сумма которых равна 2, а произведение которых равно 4. Убедиться в этом можно, вычислив дискриминант:

D1 = k− ac = (−1)− 1 × 4 = −3

А значит записывать выражение Теорема Виета рисунок 61 не имеет смысла.

Теорема Виета полезна тем, что позволяет до начала решения узнать знаки корней уравнения.

Например, запишем для уравнения x− 5+ 6 = 0 сумму и произведение его корней. Сумма корней равна второму коэффициенту, взятому с противоположным знаком, а произведение корней равно свободному члену:

Теорема Виета рисунок 50

Посмотрев на эти два равенства можно сразу понять, что оба корня должны быть положительными. Потому что произведение x× x= 6 будет выполняться только в двух случаях: если значения x1 и x2 положительны либо они оба отрицательны. Если эти значения будут отрицательными, то не будет выполняться равенство xx= 5, поскольку его правая часть равна положительному числу. А значения x1 и x2 должны удовлетворять как равенству xx= 5, так и равенству x× x= 6.

Ещё одна польза от теоремы Виета в том, что корни можно найти методом подбора. В данном примере корни должны быть такими, чтобы они удовлетворяли как равенству xx= 5 так и равенству x× x= 6. Очевидно, что таковыми являются корни 3 и 2

Теорема Виета рисунок 51

Значит, x= 3, x= 2

Теорема Виета рисунок 63


Доказательство теоремы Виета

Пусть дано приведённое квадратное уравнение xbx = 0. Если его дискриминант больше нуля, то оно имеет два корня, сумма которых равна второму коэффициенту, взятому с противоположным знаком, а произведение корней равно свободному члену:

Теорема Виета рисунок 23

Докажем, что равенства xx= −b и x× xc имеют место быть.

Вспомним формулы корней квадратного уравнения:

квадратное уравнение рисунок 90

Найдём сумму корней x1 и x2. Для этого подставим в выражение xx2 вместо x1 и x2 соответствующие выражения из правой части формул корней квадратного уравнения. Не забываем, что в приведённом квадратном уравнении xbx = 0 старший коэффициент a равен единице. Тогда в процессе подстановки знаменатель станет равен просто 2

Теорема Виета рисунок 3

Запишем правую часть в виде дроби с одним знаменателем:

Теорема Виета рисунок 5

Раскроем скобки в числителе и приведём подобные члены:

Теорема Виета рисунок 6

Сократим дробь Теорема Виета рисунок 7 на 2, тогда получим b

Теорема Виета рисунок 8

Значит xx2 действительно равно b

xx= −b

Теперь аналогично докажем, что произведение x× x2 равно свободному члену c.

Подставим вместо x1 и x2 соответствующие выражения из формул корней квадратного уравнения. Не забываем, что коэффициент a всё ещё равен единице:

Теорема Виета рисунок 10

Чтобы перемнóжить дроби, нужно перемнóжить их числители и знаменатели:

Теорема Виета рисунок 13

В числителе теперь содержится произведение суммы двух выражений и разности этих же выражений. Воспользуемся тождеством (a + b)(a − b) = a− b2. Тогда в числителе полýчится Теорема Виета рисунок 14 А знаменатель будет равен 4

Теорема Виета рисунок 16

Теперь в числителе выражение (−b)2 станет равно b2, а выражение Теорема Виета рисунок 17 станет равно просто D

Теорема Виета рисунок 15

Но D равно b− 4ac. Подстáвим это выражение вместо D, не забывая что = 1. То есть вместо b− 4ac надо подставить b− 4c

Теорема Виета рисунок 18

В получившемся выражении раскроем скобки в числителе и приведём подобные члены:

Теорема Виета рисунок 20

Сократим получившуюся дробь на 4

Теорема Виета рисунок 21

Значит x× x2 действительно равно c.

x× xc

Таким образом, сумма корней приведённого квадратного уравнения xbx = 0 равна второму коэффициенту, взятому с противоположным знáком (xx= −b), а произведение корней равно свободному члену (x× xc). Теорема доказана.


Теорема, обратная теореме Виета

Когда записана сумма и произведение корней приведённого квадратного уравнения, обычно начинается подбор подходящих корней к этому уравнению. В этот момент в работу включается так называемая теорема, обратная теореме Виета. Она формулируется так:

Если числа x1 и x2 таковы, что их сумма равна второму коэффициенту уравнения xbx = 0, взятому с противоположным знáком, а произведение чисел x1 и x2 равно свободному члену уравнения xbx = 0, то числа x1 и x2 являются корнями уравнения xbx = 0.

Обратные теоремы бывают поставлены так, что их утверждением является заключение первой теоремы.

Так, доказывая теорему Виета мы пришли к заключению, что сумма x1 и x2 равна b, а произведение x1 и x2 равно c. В обратной же теореме это заключение служит утверждением.

Ранее мы решили уравнение x− 5+ 6 = 0 и написали для него такую сумму и произведение корней:

Теорема Виета рисунок 50

А затем подобрали корни 3 и 2. По сути мы применили теорему, обратную теореме Виета. Числа 3 и 2 таковы, что их сумма равна второму коэффициенту уравнения x− 5+ 6 = 0, взятому с противоположным знаком (числу 5), а произведение чисел 3 и 2 равно свободному члену (числу 6). Значит числа 3 и 2 являются корнями уравнения x− 5+ 6 = 0.


Пример 2. Решить квадратное уравнение x− 6+ 8 = 0 по теореме, обратной теореме Виета.

В данном уравнении = 1. Значит квадратное уравнение является приведённым. Его можно решить по теореме, обратной теореме Виета.

Сначала запишем сумму и произведение корней уравнения. Сумма корней будет равна 6, поскольку второй коэффициент исходного уравнения равен −6. А произведение корней будет равно 8

Теорема Виета рисунок 26

Теперь имея эти два равенства можно подобрать подходящие корни. Они должны удовлетворять как равенству xx= 6, так и равенству x× x= 8

Подбор корней удобнее выполнять с помощью их произведения. Используя равенство x× x= 8 нужно найти такие x1 и x2, произведение которых равно 8.

Число 8 можно получить если перемножить числа 4 и 2 либо 1 и 8.

4 × 2 = 8
1 × 8 = 8

Но значения x1 и x2 надо подбирать так, чтобы они удовлетворяли не только равенству x× x= 8, но и равенству xx= 6.

Сразу делаем вывод, что значения 1 и 8 не годятся, поскольку они хоть и удовлетворяют равенству x× x= 8, но не удовлетворяют равенству xx= 6.

Зато значения 4 и 2 подходят как равенству x× x= 8, так и равенству xx= 6, поскольку эти значения удовлетворяют обоим равенствам:

Теорема Виета рисунок 28

Значит корнями уравнения x− 6+ 8 = 0 являются числа 4 и 2.

Теорема Виета рисунок 30

Обратная теорема, как и любая теорема нуждается в доказательстве. Докажем теорему, обратную теореме Виета. Для удобства корни x1 и x2 обозначим как m и n. Тогда утверждение теоремы, обратной теореме Виета примет следующий вид:

Если числа m и n таковы, что их сумма равна второму коэффициенту уравнения xbx = 0, взятому с противоположным знáком, а произведение чисел m и n равно свободному члену уравнения xbx = 0, то числа m и n являются корнями уравнения xbx = 0

Для начала запишем, что сумма m и n равна −b, а произведение mn равно c

Теорема Виета рисунок 46

Чтобы доказать, что числа m и n являются корнями уравнения xbx = 0, нужно поочередно подстáвить буквы m и n в это уравнение вместо x, затем выполнить возможные тождественные преобразования. Если в результате преобразований левая часть станет равна нулю, то это будет означать, что числа m и n являются корнями уравнения xbx = 0.

Помимо букв m и n нам нужно знать чему равен параметр b. Выразим его из равенства m + n = −b. Легче всего это сделать, умножив обе части этого равенства на −1

Теорема Виета рисунок 49

Теперь всё готово для подстановок. Подстáвим m в уравнение xbx = 0 вместо x, а выражение −m − n подставим вместо b

Теорема Виета рисунок 47

Видим, что при x = m получается верное равенство. Значит число m является корнем уравнения xbx = 0.

Аналогично докажем, что число n является корнем уравнения xbx = 0. Подставим вместо x букву n, а вместо c подставим mn, поскольку c = mn.

Теорема Виета рисунок 48

Видим, что при x = n тоже получается верное равенство. Значит число n является корнем уравнения.

Следовательно, числа m и n являются корнями уравнения xbx = 0.


Примеры решения уравнений по теореме, обратной теореме Виета

Пример 1. Решить квадратное уравнение x− 4+ 4 = 0 по теореме, обратной теореме Виета.

Запишем сумму корней x1 и x2 и приравняем её к второму коэффициенту, взятому с противоположным знаком. Также запишем произведение корней x1 и x2 и приравняем его к свободному члену:

Теорема Виета рисунок 31

В данном примере очевидно, что корнями являются числа 2 и 2. Потому что их сумма равна 4 и произведение равно 4

Теорема Виета рисунок 32

Значение x1 совпадает с x2. Это тот случай, когда квадратное уравнение имеет только один корень. Если мы попробуем решить данное уравнение с помощью формул корней квадратного уравнения, то обнаружим что дискриминант равен нулю, и корень вычисляется по формуле квадратное уравнение рисунок 96

Теорема Виета рисунок 33

Данный пример показывает, что теорема обратная теореме Виета, работает и для уравнений, имеющих только один корень. Признаком того, что квадратное уравнение имеет только один корень является то, что значения x1 и x2 совпадают.


Пример 2. Решить уравнение x+ 3+ 2 = 0 по теореме, обратной теореме Виета.

Запишем сумму и произведение корней данного уравнения:

Теорема Виета рисунок 41

Теперь подберём значения x1 и x2. Здесь начинается самое интересное. Произведение корней равно 2. Число 2 можно получить перемножив 1 и 2. Но сумма корней xx2 равна отрицательному числу −3. Значит значения 1 и 2 не подходят.

Сумма бывает отрицательной если оба слагаемых отрицательны либо отрицательным является одно слагаемое, модуль которого больше.

Если подберём корни с разными знаками, то не будет выполняться равенство x× x= 2.

Если подберем положительные корни, то будет выполняться равенство x× x= 2, но не будет выполняться равенство xx= −3.

Очевидно, что корнями являются два отрицательных числа. Произведение отрицательных чисел есть положительное число. А сумма отрицательных чисел есть отрицательное число.

Тогда равенствам будут удовлетворять числа −1 и −2.

Теорема Виета рисунок 42

Итак, корнями являются числа −1 и −2

Теорема Виета рисунок 43


Пример 3. Решить уравнение x+ 16+ 15 = 0 по теореме, обратной теореме Виета.

Запишем сумму и произведение корней данного уравнения:

Теорема Виета рисунок 44

Как и в прошлом примере сумма корней равна отрицательному числу, а произведение корней — положительному числу.

Произведение бывает положительным если оба сомножителя положительны либо оба сомножителя отрицательны. Первый вариант отпадает сразу, поскольку сумма корней равна отрицательному числу. Тогда получается, что оба корня будут отрицательными. Попробуем подобрать их.

Число 15 можно получить, если перемножить числа −1 и −15 или (−3) и (−5). В данном случае подходит первый вариант, поскольку сумма чисел −1 и −15 равна −16, а их произведение равно 15. Значит корнями уравнения x+ 16+ 15 = 0 являются числа −1 и −15

Теорема Виета рисунок 45


Пример 4. Решить уравнение x− 10− 39 = 0 по теореме, обратной теореме Виета.

Запишем сумму и произведение корней данного уравнения:

Теорема Виета рисунок 52

Произведение корней равно отрицательному числу. Значит один из корней является отрицательным. Число −39 можно получить если перемножить числа −3 и 13 либо −13 и 3. Из этих комбинаций больше годится комбинация −3 и 13, поскольку при перемножении этих чисел получается −39, а при сложении 10

Теорема Виета рисунок 53

Значит корнями уравнения x− 10− 39 = 0 являются числа −3 и 13

Теорема Виета рисунок 73


Пример 5. Первый корень уравнения xbx + 45 = 0 равен 15. Найти второй корень этого уравнения, а также значение коэффициента b.

По теореме Виета произведение корней приведённого квадратного уравнения равно свободному члену. В данном случае это произведение равно 45

x1 × x2 = 45

При этом один из корней уже известен — это корень 15.

15 × x2 = 45

Тогда второй корень будет равен 3, потому что число 45 получается, если 15 умножить на 3

15 × 3 = 45

Значит x2 = 3

Этот второй корень также можно было бы получить, выразив из равенства 15 × x2 = 45 переменную x2

Теорема Виета рисунок 74

Теперь определим значение коэффициента b. Для этого напишем сумму корней уравнения:

15 + 3 = 18

По теореме Виета сумма корней приведенного квадратного уравнения равна второму коэффициенту, взятому с противоположным знаком. Если сумма корней равна 18, а 18 это положительное число, то в самóм уравнении этот коэффициент будет отрицательным:

x2 − 18+ 45 = 0

Значит = −18.

Обычно решение к такой задаче записывают так. Сначала записывают основную теорему Виета в виде суммы и произведения корней:

Теорема Виета рисунок 23

Затем в это выражение подставляют имеющиеся известные значения. В нашем случае известно, что первый корень равен 15, а свободный член уравнения xbx + 45 = 0 равен 45

Теорема Виета рисунок 77

Из этой системы следует найти x2 и b. Выразим эти параметры:

Теорема Виета рисунок 78

Из этой системы мы видим, что x2 равно 3. Подставим его в первое равенство:

Теорема Виета рисунок 79

Теперь из первого равенства мы видим, что −b равно 18

Теорема Виета рисунок 80

Но нас интересует b, а не −b. Следует помнить, что −b это −1b. Чтобы найти b нужно 18 разделить на −1. Тогда b станет равно −18

Теорема Виета рисунок 81

Этот же результат можно получить если в выражении Теорема Виета рисунок 80 умножить первое равенство на −1

Теорема Виета рисунок 82

Теперь возвращаемся к исходному уравнению xbx + 45 = 0 и подставляем найденное значение b

Теорема Виета рисунок 83

Выполним умножение −18 на x. Получим −18x

Теорема Виета рисунок 84

Раскроем скобки:

Теорема Виета рисунок 85


Пример 6. Используя теорему Виета, написать приведённое квадратное уравнение, корнями которых являются числа 2 и 8.

В этом задании корни уже известны. То есть x= 2, x= 8. По ним надо составить квадратное уравнение вида xbx = 0.

Запишем сумму и произведение корней:

Теорема Виета рисунок 64

По теореме Виета сумма корней приведённого квадратного уравнения равна второму коэффициенту, взятому с противоположным знаком. Если сумма корней 2 и 8 равна 10, то в самóм уравнении число 10 должно быть с противоположным знаком. Значит = −10.

Произведение корней по теореме Виета равно свободному члену. У нас это произведение равно 16.

Значит = −10, = 16. Отсюда:

x2 − 10+ 16 = 0


Пример 7. Используя теорему Виета, написать приведённое квадратное уравнение, корнями которых являются числа Теорема Виета рисунок 75 и Теорема Виета рисунок 76.

Запишем сумму и произведение корней:

Теорема Виета рисунок 65

Сумма корней равна 2. Тогда в уравнении второй коэффициент будет равен −2. А произведение корней равно −1. Значит свободный член будет равен −1. Тогда:

x2 − 2x − 1 = 0


Когда квадратное уравнение неприведённое

Теорема Виета выполняется только тогда, когда квадратное уравнение является приведённым.

Если квадратное уравнение не является приведённым, но всё равно возникла необходимость применить теорему Виета, то обе части неприведённого квадратного уравнения следует разделить на коэффициент, который располагается перед x2.

Если к примеру в квадратном уравнении axbx = 0 коэффициент a не равен единице, то данное уравнение является неприведённым. Чтобы сделать его приведённым, надо разделить обе его части на коэффициент, который располагается перед x2, то есть на a

Теорема Виета рисунок 67

Получилось уравнение приведенное квадратное уравнение, которое является приведённым. В нём второй коэффициент равен b na a, а свободный член равен c na a. Тогда сумма и произведение корней будут выглядеть так:

Теорема Виета рисунок 66

Например, решим квадратное уравнение 4x+ 5+ 1 = 0. Это уравнение не является приведённым. Приведённым оно станет, если разделить обе его части на коэффициент, который располагается перед x2, то есть на 4

теорема виета рисунок 68

Получили приведённое квадратное уравнение. В нём второй коэффициент равен 5 на 4, а свободный член одна четвертая. Тогда по теореме Виета имеем:

Теорема Виета рисунок 66

Отсюда методом подбора находим корни −1 и

Теорема Виета рисунок 67

Возможно этот метод вы редко будете использовать при решении квадратных уравнений. Но знать о нём не помешает.


 

Пример 2. Решить квадратное уравнение 3x− 7+ 2 = 0

Данное уравнение не является приведённым, а значит его пока нельзя решить по теореме, обратной теореме Виета.

Сделаем данное уравнение приведенным. Разделим обе части на коэффициент, который располагается перед x2

Теорема Виета рисунок 36

Получили уравнение Теорема Виета рисунок 36. Запишем сумму и произведение корней этого уравнения:

Теорема Виета рисунок 37

Отсюда методом подбора находим корни 2 и одна третья

Теорема Виета рисунок 40


Пример 3. Решить квадратное уравнение 2x− 3− 2 = 0

Это неприведённое квадратное уравнение. Чтобы сделать его приведённым, нужно разделить обе его части на 2. Сделать это можно в уме. Если 2x2 разделить на 2, то полýчится x2

икс в квадрате строчное выражение

Далее если −3x разделить на 2, то полýчится минус 3x на 2. Чтобы видеть где коэффициент, а где переменная, такое выражение записывают в виде минус 3x на 2 2

теорема виета рисунок 69

Далее если −2 разделить на 2, то полýчится −1

теорема виета рисунок 71

Прирáвниваем получившееся выражение к нулю:

теорема виета рисунок 70

Теперь применяем теорему Виета. Сумма корней будет равна второму коэффициенту, взятому с противоположным знáком, а произведение корней свободному члену:

теорема виета рисунок 72

Отсюда методом подбора находим корни 2 и минус одна вторая


Задания для самостоятельного решения

Задание 1. Написать сумму и произведение корней для квадратного уравнения:
Решение:
Задание 2. Написать сумму и произведение корней для квадратного уравнения:
Решение:
Задание 3. Написать сумму и произведение корней для квадратного уравнения:
Решение:
Задание 4. Решить квадратное уравнение по теореме, обратной теореме Виета:
Решение:
Задание 5. Решить квадратное уравнение по теореме, обратной теореме Виета:
Решение:
Задание 6. Решить квадратное уравнение по теореме, обратной теореме Виета:
Решение:
Задание 7. Решить квадратное уравнение по теореме, обратной теореме Виета:
Решение:
Задание 8. Решить квадратное уравнение по теореме, обратной теореме Виета:
Решение:
Задание 9. Решить квадратное уравнение по теореме, обратной теореме Виета:
Решение:

Понравился урок?
Вступай в нашу новую группу Вконтакте и начни получать уведомления о новых уроках

Возникло желание поддержать проект?
Используй кнопку ниже

Квадратное уравнение с чётным вторым коэффициентом

Если в квадратном уравнении axbx = 0 второй коэффициент b является чётным, то решение этого уравнения можно немного упростить. Дискриминант для такого уравнения можно вычислить по формуле Dk− ac, а корни по формулам квадратное уравнение с четным коэффициентом рисунок 1 и квадратное уравнение с четным коэффициентом рисунок 2.

Примеры

Решим квадратное уравнение x+ 6− 16 = 0. В нём второй коэффициент является чётным. Чтобы воспользоваться формулами для чётного коэффициента, нужно сначала узнать чему равна переменная k.

Любое четное число n можно представить в виде произведения числа 2 и числа k, то есть 2k.

n = 2k

Например, число 10 можно представить как 2 × 5.

10 = 2 × 5

В этом произведении = 5.


Число 12 можно представить как 2 × 6.

12 = 2 × 6

В этом произведении = 6.


Число −14 можно представить как 2 × (−7)

В этом произведении = −7.

Как видим, сомножитель 2 не меняется. Меняется только сомножитель k.

В уравнении x+ 6x − 16 = 0 вторым коэффициентом является число 6. Это число можно представить как 2 × 3. В этом произведении = 3. Теперь можно воспользоваться формулами для чётного коэффициента.

Найдем дискриминант по формуле Dk− ac

Dk− ac = 3− 1 × (−16) = 9 + 16 = 25

Теперь вычислим корни по формулам: квадратное уравнение с четным коэффициентом рисунок 1 и квадратное уравнение с четным коэффициентом рисунок 2.

квадратное уравнение с четным коэффициентом рисунок 3

Значит корнями уравнения x+ 6x − 16 = 0 являются числа 2 и −8.

В отличие от стандартной формулы для вычисления дискриминанта (D=b− 4ac), в формуле Dk− ac не нужно выполнять умножение числа 4 на ac.

И в отличие от формул формула для вычисления первого корня квадратного уравнения и формула для вычисления второго корня квадратного уравнения формулы квадратное уравнение с четным коэффициентом рисунок 1 и квадратное уравнение с четным коэффициентом рисунок 2 не содержат в знаменателе множитель 2 что опять же освобождает нас от дополнительных вычислений.


Пример 2. Решить квадратное уравнение 5x− 6+ 1=0

Второй коэффициент является чётным числом. Его можно представить в виде 2 × (−3). То есть = −3. Найдём дискриминант по формуле Dk− ac

Dk− ac = (−3)− 5 × 1 = 9 − 5 = 4

Дискриминант больше нуля. Значит уравнение имеет два корня. Для их вычисления воспользуемся формулами квадратное уравнение с четным коэффициентом рисунок 1 и квадратное уравнение с четным коэффициентом рисунок 2

квадратное уравнение с четным коэффициентом рисунок 11


Пример 3. Решить квадратное уравнение x− 10− 24 = 0

Второй коэффициент является чётным числом. Его можно представить в виде 2 × (−5). То есть = −5. Найдём дискриминант по формуле Dk− ac

Dk− ac = (−5)− 1 × (−24) = 25 + 24 = 49

Дискриминант больше нуля. Значит уравнение имеет два корня. Для их вычисления воспользуемся формулами квадратное уравнение с четным коэффициентом рисунок 1 и квадратное уравнение с четным коэффициентом рисунок 2

квадратное уравнение с четным коэффициентом рисунок 12

Обычно для определения числа k поступают так: делят второй коэффициент на 2.

Действительно, если второй коэффициент b является чётным числом, то его можно представить как b = 2k. Чтобы из этого равенства выразить сомножитель k, нужно произведение b разделить на сомножитель 2

квадратное уравнение с четным коэффициентом рисунок 14

Например, в предыдущем примере для определения числа k можно было просто разделить второй коэффициент −10 на 2

квадратное уравнение с четным коэффициентом рисунок 15


Пример 5. Решить квадратное уравнение квадратное уравнение с чётным вторым коэффициентом рисунок 16

Коэффициент b равен два корня из двух. Это выражение состоит из множителя 2 и выражения корень из 2. То есть оно уже представлено в виде 2k. Получается, что k равно два корня из двух

Найдём дискриминант по формуле Dk− ac

квадратное уравнение с чётным вторым коэффициентом рисунок 17

Дискриминант больше нуля. Значит уравнение имеет два корня. Для их вычисления воспользуемся формулами квадратное уравнение с четным коэффициентом рисунок 1 и квадратное уравнение с четным коэффициентом рисунок 2

квадратное уравнение с чётным вторым коэффициентом рисунок 18

При вычислении корня уравнения получилась дробь, в которой содержится квадратный корень из числа 2. Квадратный корень из числа 2 извлекается только приближённо. Если выполнить это приближённое извлечение, а затем сложить результат с 2, и затем разделить числитель на знаменатель, то получится не очень красивый ответ.

В таких случаях ответ записывают, не выполняя приближённых вычислений. В нашем случае первый корень уравнения будет равен квадратное уравнение с чётным вторым коэффициентом рисунок 19.

Вычислим второй корень уравнения:

квадратное уравнение с чётным вторым коэффициентом рисунок 20


Вывод формул

Давайте наглядно увидим, как появились формулы для вычисления корней квадратного уравнения с чётным вторым коэффициентом.

Рассмотрим квадратное уравнение axbx = 0. Допустим, что коэффициент b является чётным числом. Тогда его можно обозначить как 2k

b = 2k

Заменим в уравнении axbx = 0 коэффициент b на выражение 2k

ax+ 2kx = 0

Теперь вычислим дискриминант по ранее известной формуле:

D = b− 4ac = (2k)4ac = 4k− 4ac

Вынесем в получившемся выражении за скобки общий множитель 4

D = b− 4ac = (2k)2 − 4ac = 4k− 4ac = 4(k− ac)

Что можно сказать о получившемся дискриминанте? При чётном втором коэффициенте он состоит из множителя 4 и выражения k− ac.

В выражении 4(k− ac) множитель 4 постоянен. Значит знак дискриминанта зависит от выражения k− ac. Если это выражение меньше нуля, то и D будет меньше нуля. Если это выражение больше нуля, то и D будет больше нуля. Если это выражение равно нулю, то и D будет равно нулю.

То есть выражение k− ac это различитель — дискриминант. Такой дискриминант принято обозначать буквой D1

Dk− ac

Теперь посмотрим как выводятся формулы квадратное уравнение с четным коэффициентом рисунок 1 и квадратное уравнение с четным коэффициентом рисунок 2.

В нашем уравнении axbx = 0 коэффициент b заменён на выражение 2k. Воспользуемся стандартными формулами для вычисления корней. То есть формулами формула для вычисления первого корня квадратного уравнения и формула для вычисления второго корня квадратного уравнения. Только вместо b будем подставлять 2k. Также на забываем, что D у нас равно выражению 4(k− ac)

квадратное уравнение с четным коэффициентом рисунок 5

Но ранее было сказано, что выражение k− ac обозначается через D1. Тогда в наших преобразованиях следует сделать и эту замену:

квадратное уравнение с четным коэффициентом рисунок 6

Теперь вычислим квадратный корень, расположенный в числителе. Это квадратный корень из произведения — он равен произведению корней. Остальное перепишем без изменений:

квадратное уравнение с четным коэффициентом рисунок 7

Теперь в получившемся выражении вынесем за скобки общий множитель 2

квадратное уравнение с четным коэффициентом рисунок 8

Сократим получившуюся дробь на 2

квадратное уравнение с четным коэффициентом рисунок 9

Аналогично вывóдится формула для вычисления второго корня:

квадратное уравнение с четным коэффициентом рисунок 10

Задания для самостоятельного решения

Задание 1. Решить уравнение:
Решение:
Ответ: 1; 0,6
Задание 2. Решить уравнение:
Решение:
Ответ:
Задание 3. Решить уравнение:
Решение:
Ответ: 1; −1,4
Задание 4. Решить уравнение:
Решение:
Ответ:
Задание 5. Решить уравнение:
Решение:
Ответ:
Задание 6. Решить уравнение:
Решение:
Ответ:
Задание 7. Решить уравнение:
Решение:
Ответ:

Понравился урок?
Вступай в нашу новую группу Вконтакте и начни получать уведомления о новых уроках

Возникло желание поддержать проект?
Используй кнопку ниже

Квадратное уравнение

Что такое квадратное уравнение и как его решать?

Мы помним, что уравнение это равенство, содержащее в себе переменную, значение которой нужно найти.

Если переменная, входящая в уравнение, возведенá во вторую степень (в квадрат), то такое уравнение называют уравнением второй степени или квадратным уравнением.

Например, следующие уравнения являются квадратными:

примеры квадратных уравнений

Решим первое из этих уравнений, а именно x− 4 = 0.

Все тождественные преобразования, которые мы применяли при решении обычных линейных уравнений, можно применять и при решении квадратных.

Итак,  в уравнении x− 4 = 0 перенесем член −4 из левой части в правую часть, изменив знак:

квадратное уравнение рисунок 120

Получили уравнение x= 4. Ранее мы говорили, что уравнение считается решённым, если в одной части переменная записана в первой степени и её коэффициент равен единице, а другая часть равна какому-нибудь числу. То есть чтобы решить уравнение, его следует привести к виду x = a, где a — корень уравнения.

У нас переменная x всё ещё во второй степени, поэтому решение необходимо продолжить.

Чтобы решить уравнение x= 4, нужно ответить на вопрос при каком значении x левая часть станет равна 4. Очевидно, что при значениях 2 и −2. Чтобы вывести эти значения воспользуемся определением квадратного корня.

Число b называется квадратным корнем из числа a, если b= a и обозначается как b равно корень из a

У нас сейчас похожая ситуация. Ведь, что такое x= 4? Переменная x в данном случае это квадратный корень из числа 4, поскольку вторая степень x прирáвнена к 4.

Тогда можно записать, что x равно корень из 4. Вычисление правой части позвóлит узнать чему равно x. Квадратный корень имеет два значения: положительное и отрицательное. Тогда получаем = 2 и = −2.

Обычно записывают так: перед квадратным корнем ставят знак «плюс-минус», затем находят арифметическое значение квадратного корня. В нашем случае на этапе когда записано выражение x равно корень из 4, перед корень кв из 4 следует поставить знак ±

квадратное уравнение рисунок 35

Затем найти арифметическое значение квадратного корня корень кв из 4

квадратное уравнение рисунок 36

Выражение = ± 2 означает, что = 2 и = −2. То есть корнями уравнения x− 4 = 0 являются числа 2 и −2. Запишем полностью решение данного уравнения:

квадратное уравнение рисунок 9

Выполним проверку. Подставим корни 2 и −2 в исходное уравнение и выполним соответствующие вычисления. Если при значениях 2 и −2 левая часть равна нулю, то это будет означать, что уравнение решено верно:

квадратное уравнение рисунок 10

В обоих случаях левая часть равна нулю. Значит уравнение решено верно.

Решим ещё одно уравнение. Пусть требуется решить квадратное уравнение (+ 2)= 25

Для начала проанализируем данное уравнение. Левая часть возведенá в квадрат и она равна 25. Какое число в квадрате равно 25? Очевидно, что числа 5 и −5

квадратное уравнение рисунок 24

То есть наша задача найти x, при которых выражение + 2 будет равно числам 5 и −5. Запишем эти два уравнения:

квадратное уравнение рисунок 12

Решим оба уравнения. Это обычные линейные уравнения, которые решаются легко:

квадратное уравнение рисунок 11

Значит корнями уравнения (+ 2)= 25 являются числа 3 и −7.

В данном примере как и в прошлом можно использовать определение квадратного корня. Так, в уравнения (+ 2)= 25 выражение (+ 2) представляет собой квадратный корень из числа 25. Поэтому можно cначала записать, что квадратное уравнение рисунок 37.

Тогда правая часть станет равна ±5. Полýчится два уравнения: + 2 = 5 и + 2 = −5. Решив по отдельности каждое из этих уравнений мы придём к корням 3 и −7.

Запишем полностью решение уравнения (+ 2)= 25

квадратное уравнение рисунок 38

Из рассмотренных примеров видно, что квадратное уравнение имеет два корня. Чтобы не забыть о найденных корнях, переменную x можно подписывать нижними индексами. Так, корень 3 можно обозначить через x1, а корень −7 через x2

квадратное уравнение рисунок 13

В предыдущем примере тоже можно было сделать так. Уравнение x− 4 = 0 имело корни 2 и −2. Эти корни можно было обозначить как x= 2 и x= −2. 

Бывает и так, что квадратное уравнение имеет только один корень или вовсе не имеет корней. Такие уравнения мы рассмотрим позже.

Сделаем проверку для уравнения (+ 2)= 25. Подставим в него корни 3 и −7. Если при значениях 3 и −7 левая часть равна 25, то это будет означать, что уравнение решено верно:

квадратное уравнение рисунок 14

В обоих случаях левая часть равна 25. Значит уравнение решено верно.

Квадратное уравнение бывает дано в разном виде. Наиболее его распространенная форма выглядит так:

ax2 + bx + c = 0,
где a, b, c — некоторые числа, x — неизвестное.

Это так называемый общий вид квадратного уравнения. В таком уравнении все члены собраны в общем месте (в одной части), а другая часть равна нулю. По другому такой вид уравнения называют нормальным видом квадратного уравнения.

Пусть дано уравнение 3x+ 2= 16. В нём переменная x возведенá во вторую степень, значит уравнение является квадратным. Приведём данное уравнение к общему виду.

Итак, нам нужно получить уравнение, которое будет похоже на уравнение axbx = 0. Для этого в уравнении 3x+ 2= 16 перенесем 16 из правой части в левую часть, изменив знак:

3x2 + 2x − 16 = 0

Получили уравнение 3x+ 2− 16 = 0. В этом уравнении = 3, = 2, = −16.

В квадратном уравнении вида axbx = 0 числа a, b и c имеют собственные названия. Так, число a называют первым или старшим коэффициентом; число b называют вторым коэффициентом; число c называют свободным членом.

В нашем случае для уравнения 3x+ 2− 16 = 0 первым или старшим коэффициентом является 3; вторым коэффициентом является число 2;  свободным членом является число −16. Есть ещё другое общее название для чисел a, b и c — параметры.

Так, в уравнении 3x+ 2− 16 = 0 параметрами являются числа 3, 2 и −16.

В квадратном уравнении желательно упорядочивать члены так, чтобы они располагались в таком же порядке как у нормального вида квадратного уравнения.

Например, если дано уравнение −5 + 4x= 0, то его желательно записать в нормальном виде, то есть в виде ax2+ bx + c = 0.

В уравнении −5 + 4xx = 0 видно, что свободным членом является −5, он должен располагаться в конце левой части. Член 4x2 содержит старший коэффициент, он должен располагаться первым. Член x соответственно будет располагаться вторым:

квадратное уравнение рисунок 39

Квадратное уравнение в зависимости от случая может принимать различный вид. Всё зависит от того, чему равны значения a, b и с.

Если коэффициенты a, b и c не равны нулю, то квадратное уравнение называют полным. Например, полным является квадратное уравнение 2x+ 6x − 8 = 0.

Если какой-то из коэффициентов равен нулю (то есть отсутствует), то уравнение значительно уменьшается и принимает более простой вид. Такое квадратное уравнение называют неполным. Например, неполным является квадратное уравнение 2x+ 6= 0, в нём имеются коэффициенты a и b (числа 2 и 6), но отсутствует свободный член c.

Рассмотрим каждый из этих видов уравнений, и для каждого из этих видов определим свой способ решения.

Пусть дано квадратное уравнение 2x+ 6x − 8 = 0. В этом уравнении = 2, = 6, = −8. Если b сделать равным нулю, то уравнение примет вид:

квадратное уравнение рисунок 3

Получилось уравнение 2x− 8 = 0. Чтобы его решить перенесем −8 в правую часть, изменив знак:

2x= 8

Для дальнейшего упрощения уравнения воспользуемся ранее изученными тождественными преобразованиями. В данном случае можно разделить обе части на 2

квадратное уравнение рисунок 2

У нас получилось уравнение, которое мы решали в начале данного урока. Чтобы решить уравнение x= 4, следует воспользоваться определением квадратного корня. Если x= 4, то квадратное уравнение рисунок 40. Отсюда = 2 и = −2.

Значит корнями уравнения 2x− 8 = 0 являются числа 2 и −2. Запишем полностью решение данного уравнения:

квадратное уравнение рисунок 1

Выполним проверку. Подставим корни 2 и −2 в исходное уравнение и выполним соответствующие вычисления. Если при значениях 2 и −2 левая часть равна нулю, то это будет означать, что уравнение решено верно:

квадратное уравнение рисунок 4

В обоих случаях левая часть равна нулю, значит уравнение решено верно.

Уравнение, которое мы сейчас решили, является неполным квадратным уравнением. Название говорит само за себя. Если полное квадратное уравнение выглядит как axbx = 0, то сделав коэффициент b нулём получится неполное квадратное уравнение ax= 0.

У нас тоже сначала было полное квадратное уравнение 2x+ 6− 4 = 0. Но мы сделали коэффициент b нулем, то есть вместо числа 6 поставили 0. В результате уравнение обратилось в неполное квадратное уравнение 2x− 4 = 0.

В начале данного урока мы решили квадратное уравнение x− 4 = 0. Оно тоже является уравнением вида ax= 0, то есть неполным. В нем = 1, = 0, с = −4.

Также, неполным будет квадратное уравнение, если коэффициент c равен нулю.

Рассмотрим полное квадратное уравнение 2x+ 6x − 4 = 0. Сделаем коэффициент c нулём. То есть вместо числа 4 поставим 0

квадратное уравнение рисунок 25

Получили квадратное уравнение 2x+ 6x=0, которое является неполным. Чтобы решить такое уравнение, переменную x выносят за скобки:

квадратное уравнение рисунок 26

Получилось уравнение x(2+ 6) = 0 в котором нужно найти x, при котором левая часть станет равна нулю. Заметим, что в этом уравнении выражения x и (2+ 6) являются сомножителями. Одно из свойств умножения говорит, что произведение равно нулю, если хотя бы один из сомножителей равен нулю (или первый сомножитель или второй).

В нашем случае равенство будет достигаться, если x будет равно нулю или (2+ 6) будет равно нулю. Так и запишем для начала:

квадратное уравнение рисунок 6

Получилось два уравнения: = 0 и 2+ 6 = 0. Первое уравнение решать не нужно — оно уже решено. То есть первый корень равен нулю.

Чтобы найти второй корень, решим уравнение 2+ 6 = 0. Это обычное линейное уравнение, которое решается легко:

квадратное уравнение рисунок 5

Видим, что второй корень равен −3.

Значит корнями уравнения 2x+ 6= 0 являются числа 0 и −3. Запишем полностью решение данного уравнения:

квадратное уравнение рисунок 7

Выполним проверку. Подставим корни 0 и −3 в исходное уравнение и выполним соответствующие вычисления. Если при значениях 0 и −3 левая часть равна нулю, то это будет означать, что уравнение решено верно:

квадратное уравнение рисунок 8

Следующий случай это когда числа b и с равны нулю. Рассмотрим полное квадратное уравнение 2x+ 6− 4 = 0. Сделаем коэффициенты b и c нулями. Тогда уравнение примет вид:

квадратное уравнение рисунок 27

Получили уравнение 2x= 0. Левая часть является произведением, а правая часть равна нулю. Произведение равно нулю, если хотя бы один из сомножителей равен нулю. Очевидно, что = 0. Действительно, 2 × 0= 0. Отсюда, 0 = 0. При других значениях x равенства достигаться не будет.

Проще говоря, если в квадратном уравнении вида axbx = 0 числа b и с равны нулю, то корень такого уравнения равен нулю.

Отметим, что когда употребляются словосочетания «b равно нулю» или «с равно нулю«, то подразумевается, что параметры b или c вовсе отсутствуют в уравнении.

Например, если дано уравнение 2x− 32 = 0, то мы говорим, что = 0. Потому что если сравнить с полным уравнением axbx = 0, то можно заметить, что в уравнении 2x− 32 = 0 присутствует старший коэффициент a, равный 2; присутствует свободный член −32; но отсутствует коэффициент b.

Наконец, рассмотрим полное квадратное уравнение axbx = 0. В качестве примера решим квадратное уравнение x− 2+ 1 = 0.

Итак, требуется найти x, при котором левая часть станет равна нулю. Воспользуемся изученными ранее тождественными преобразованиями.

Прежде всего заметим, что левая часть уравнения представляет собой квадрат разности двух выражений. Если мы вспомним как раскладывать многочлен на множители, то получим в левой части (− 1)2.

квадратное уравнение рисунок 41

Рассуждаем дальше. Левая часть возведенá в квадрат и она равна нулю. Какое число в квадрате равно нулю? Очевидно, что только 0. Поэтому наша задача найти x, при котором выражение − 1 равно нулю. Решив простейшее уравнение − 1 = 0, можно узнать чему равно x

квадратное уравнение рисунок 42

Этот же результат можно получить, если воспользоваться квадратным корнем. В уравнении (− 1)= 0 выражение (− 1) представляет собой квадратный корень из нуля. Тогда можно записать, что квадратное уравнение рисунок 43. В этом примере записывать перед корнем знак ± не нужно, поскольку корень из нуля имеет только одно значение — ноль. Тогда получается − 1 = 0. Отсюда = 1.

Значит корнем уравнения x− 2+ 1 = 0 является единица. Других корней у данного уравнения нет. В данном случае мы решили квадратное уравнение, имеющее только один корень. Такое тоже бывает.

Не всегда бывают даны простые уравнения. Рассмотрим например уравнение x+ 2− 3 = 0.

В данном случае левая часть уже не является квадратом суммы или разности. Поэтому нужно искать другие пути решения.

Заметим, что левая часть уравнения представляет собой квадратный трехчлен. Тогда можно попробовать выделить полный квадрат из этого трёхчлена и посмотреть что это нам даст.

Выделим полный квадрат из квадратного трёхчлена, располагающего в левой части уравнения:

квадратное уравнение рисунок 16

В получившемся уравнении перенесем −4 в правую часть, изменив знак:

квадратное уравнение рисунок 18

Теперь воспользуемся квадратным корнем. В уравнении (+ 1)= 4 выражение (+ 1) представляет собой квадратный корень из числа 4. Тогда можно записать, что квадратное уравнение рисунок 44. Вычисление правой части даст выражение + 1 = ±2. Отсюда полýчится два уравнения: + 1 = 2 и + 1 = −2, корнями которых являются числа 1 и −3

квадратное уравнение рисунок 45

Значит корнями уравнения x+ 2− 3 = 0 являются числа 1 и −3.

Выполним проверку:

квадратное уравнение рисунок 20


Пример 3. Решить уравнение x− 6+ 9 = 0, выделив полный квадрат.

Выделим полный квадрат из левой части:

квадратное уравнение рисунок 21

Далее воспользуемся квадратным корнем и узнáем чему равно x

квадратное уравнение рисунок 28

Значит корнем уравнения x− 6+ 9 = 0 является 3. Выполним проверку:

квадратное уравнение рисунок 23


Пример 4. Решить квадратное уравнение 4x+ 28− 72 = 0, выделив полный квадрат:

Выделим полный квадрат из левой части:

квадратное уравнение рисунок 29

Перенесём −121 из левой части в правую часть, изменив знак:

квадратное уравнение рисунок 30

Воспользуемся квадратным корнем:

квадратное уравнение рисунок 46

Получили два простых уравнения: 2+ 7 = 11 и 2+ 7 = −11. Решим их:

квадратное уравнение рисунок 32


Пример 5. Решить уравнение 2x+ 3− 27 = 0

Это уравнение немного посложнее. Когда мы выделяем полный квадрат, первый член квадратного трёхчлена мы представляем в виде квадрата какого-нибудь выражения.

Так, в прошлом примере первым членом уравнения был 4x2. Его можно было представить в виде квадрата выражения 2x, то есть (2x)= 22x= 4x2. Чтобы убедиться что это правильно, можно извлечь квадратный корень из выражения 4x2. Это квадратный корень из произведения — он равен произведению корней:

квадратное уравнение рисунок 121

В уравнении 2x+ 3− 27 = 0 первый член это 2x2. Его нельзя представить в виде квадрата какого-нибудь выражения. Потому что нет числá, квадрат которого равен 2. Если бы такое число было, то этим числом был бы квадратный корень из числа 2. Но квадратный корень из числа 2 извлекается только приближённо. А приближённое значение не годится для представления числá 2 в виде квадрата.

Если обе части исходного уравнения умножить или разделить на одно и то же число, то полýчится уравнение равносильное исходному. Это правило сохраняется и для квадратного уравнения.

Тогда можно разделить обе части нашего уравнения на 2. Это позвóлит избавиться от двойки перед x2 что впоследствии даст нам возможность выделить полный квадрат:

квадратное уравнение рисунок 33

Перепишем левую часть в виде трёх дробей со знаменателем 2

квадратное уравнение рисунок 34

Сократим первую дробь на 2. Остальные члены левой части перепишем без изменений. Правая часть по-прежнему станет равна нулю:

квадратное уравнение рисунок 47

Выделим полный квадрат.

квадратное уравнение рисунок 48

При представлении члена 3 на 2 на x в виде удвоенного произведения, появление множителя 2 привело бы к тому, что этот множитель и знаменатель дроби три вторых сократились бы. Чтобы этого не произошло, удвоенное произведение было домножено на одна вторая. При выделении полного квадрата всегда нужно стараться сделать так, чтобы значение изначального выражения не изменилось.

Свернём полученный полный квадрат:

квадратное уравнение рисунок 49

Приведём подобные члены:

квадратное уравнение рисунок 50

Перенесём дробь квадратное уравнение рисунок 51 в правую часть, изменив знак:

квадратное уравнение рисунок 52

Воспользуемся квадратным корнем. Выражение квадратное уравнение рисунок 53 представляет собой квадратный корень из числа квадратное уравнение рисунок 54

квадратное уравнение рисунок 55

Для вычисления правой части воспользуемся правилом извлечения квадратного корня из дроби:

квадратный корень из дроби

Тогда наше уравнение примет вид:

квадратное уравнение рисунок 56

Полýчим два уравнения:

квадратное уравнение рисунок 58

Решим их:

квадратное уравнение рисунок 59

Значит корнями уравнения 2x+ 3− 27 = 0 являются числа 3 и -9 na 2.

Корень -9 na 2 удобнее оставить в таком виде, не выполняя деления числителя на знаменатель. Так проще будет выполнять проверку.

Выполним проверку. Подставим найденные корни в исходное уравнение:

квадратное уравнение рисунок 60

В обоих случаях левая часть равна нулю, значит уравнение 2x+ 3− 27 = 0 решено верно.

Решая уравнение 2x+ 3− 27 = 0, в самом начале мы разделили обе его части на 2. В результате получили квадратное уравнение, в котором коэффициент перед x2 равен единице:

квадратное уравнение рисунок 47

Такой вид квадратного уравнения называют приведённым квадратным уравнением.

Любое квадратное уравнение вида axbx = 0 можно сделать приведённым. Для этого нужно разделить обе его части на коэффициент, который располагается перед x². В данном случае обе части уравнения axbx = 0 нужно разделить на a

квадратное уравнение рисунок 122


Пример 6. Решить квадратное уравнение 2x+ 2 = 0

Сделаем данное уравнение приведённым:

квадратное уравнение рисунок 75

Выделим полный квадрат:

квадратное уравнение рисунок 76

Получили уравнение квадратное уравнение рисунок 77 , в котором квадрат выражения квадратное уравнение рисунок 78 равен отрицательному числу квадратное уравнение рисунок 79. Такого быть не может, поскольку квадрат любого числа или выражения всегда положителен.

Следовательно, нет такого значения x, при котором левая часть стала бы равна квадратное уравнение рисунок 79. Значит уравнение квадратное уравнение рисунок 77 не имеет корней.

А поскольку уравнение квадратное уравнение рисунок 77 равносильно исходному уравнению 2x+ 2 = 0, то и оно (исходное уравнение) не имеет корней.


Формулы корней квадратного уравнения

Выделять полный квадрат для каждого решаемого квадратного уравнения не очень удобно.

Можно ли создать универсальные формулы для решения квадратных уравнений? Оказывается можно. Сейчас мы этим и займёмся.

Взяв за основу буквенное уравнение axbx = 0, и выполнив некоторые тождественные преобразования, мы сможем получить формулы для вывода корней квадратного уравнения axbx = 0. В эти формулы можно будет подставлять коэффициенты a, b, с и получать готовые решения.

Итак, выделим полный квадрат из левой части уравнения axbx = 0. Сначала сделаем данное уравнение приведённым. Разделим обе его части на a

квадратное уравнение рисунок 61

Теперь в получившемся уравнении выделим полный квадрат:

квадратное уравнение рисунок 62

Перенесем члены квадратное уравнение рисунок 64 и квадратное уравнение рисунок 65 в правую часть, изменив знак:

квадратное уравнение рисунок 66

Приведём правую часть к общему знаменателю. Дроби, состоящие из букв, привóдят к общему знаменателю методом «крест-нáкрест». То есть знаменатель первой дроби станóвится дополнительным множителем второй дроби, а знаменатель второй дроби станóвится дополнительным множителем первой дроби:

квадратное уравнение рисунок 67

В числителе правой части вынесем за скобки a

квадратное уравнение рисунок 68

Сократим правую часть на a

квадратное уравнение рисунок 69

Поскольку все преобразования были тождественными, то получившееся уравнение квадратное уравнение рисунок 74 имеет те же корни, что и исходное уравнение axbx = 0.

Уравнение квадратное уравнение рисунок 74 будет иметь корни только тогда, если правая часть больше нуля или равна нулю. Это потому что в левой части выполнено возведéние в квадрат, а квадрат любого числа положителен или равен нулю (если в этот квадрат возвóдится ноль). А чему будет равна правая часть зависит от того, что будет подставлено вместо переменных a, b и c.

Поскольку при любом a не рáвным нулю, знаменатель правой части уравнения квадратное уравнение рисунок 74 всегда будет положительным, то знак дроби квадратное уравнение рисунок 72 будет зависеть от знака её числителя, то есть от выражения b− 4ac.

Выражение b− 4ac называют дискриминантом квадратного уравнения. Дискриминант это латинское слово, означающее различитель. Дискриминант квадратного уравнения обозначается через букву D

D = b2 4ac

Дискриминант позволяет заранее узнать имеет ли уравнение корни или нет. Так, в предыдущем задании мы долго решали уравнение 2x+ 2 = 0 и оказалось, что оно не имеет корней. Дискриминант же позволил бы нам заранее узнать, что корней нет. В уравнении 2x+ 2 = 0 коэффициенты a, b и c равны 2, 1 и 2 соответственно. Подставим их в формулу D = b2−4ac

D = b2 − 4ac = 12 − 4 × 2 × 2 = 1 − 16 = −15.

Видим, что D (оно же b− 4ac) является отрицательным числом. Тогда нет смысла решать уравнение 2x+ 2 = 0, выделяя в нём полный квадрат, потому что когда мы дойдем до уравнения вида квадратное уравнение рисунок 74, окажется что правая часть станет меньше нуля (из-за отрицательного дискриминанта). А квадрат числа не может быть отрицательным. Следовательно, корней у данного уравнения не будет.

Станóвится понятно почему древние люди считали выражение b− 4ac различителем. Это выражение подобно индикатору позволяет различить уравнение имеющего корни от уравнения, не имеющего корней.

Итак, D равно b− 4ac. Подставим в уравнении квадратное уравнение рисунок 74 вместо выражения b− 4ac букву D

квадратное уравнение рисунок 80

Если дискриминант исходного уравнения окажется меньше нуля (< 0), то уравнение примет вид:

квадратное уравнение рисунок 81

В этом случае говорят, что у исходного уравнения корней нет, поскольку квадрат любого числа не должен быть отрицательным.

Если дискриминант исходного уравнения окажется больше нуля (> 0), то уравнение примет вид:

квадратное уравнение рисунок 82

В этом случае уравнение будет иметь два корня. Для их вывода воспользуемся квадратным корнем:

квадратное уравнение рисунок 83

Получили уравнение квадратное уравнение рисунок 84. Из него полýчится два уравнения: квадратное уравнение рисунок 85 и квадратное уравнение рисунок 86. Выразим x в каждом из уравнений:

квадратное уравнение рисунок 87

Получившиеся два равенства это и есть универсальные формулы для решения квадратного уравнения axbx = 0. Их называют формулами корней квадратного уравнения.

Чаще всего эти формулы обозначаются как x1 и x2. То есть для вычисления первого корня используется формула c индексом 1; для вывода второго корня — формула с индексом 2. Обозначим свои формулы так же:

квадратное уравнение рисунок 90

Очерёдность применения формул не важнá.

Решим например квадратное уравнение x+ 2− 8 = 0 с помощью формул корней квадратного уравнения. Коэффициенты данного квадратного уравнения это числа 1, 2 и −8. То есть, = 1, = 2, = −8.

Прежде чем использовать формулы корней квадратного уравнения, нужно найти дискриминант этого уравнения.

Найдём дискриминант квадратного уравнения. Для этого воспользуемся формулой D = b2 4ac. Вместо переменных a, b и c у нас будут коэффициенты уравнения x+ 2− 8 = 0

D = b2 4ac = 22− 4 × 1 × (−8) = 4 + 32 = 36

Дискриминант больше нуля. Значит уравнение имеет два корня. Теперь можно воспользоваться формулами корней квадратного уравнения:

квадратное уравнение рисунок 91

Значит корнями уравнения x+ 2− 8 = 0 являются числа 2 и −4. Проверкой убеждаемся, что корни найдены верно:

квадратное уравнение рисунок 92

Наконец, рассмотрим случай когда дискриминант квадратного уравнения равен нулю. Вернёмся к уравнению квадратное уравнение рисунок 80. Если дискриминант равен нулю, то правая часть уравнения примет вид:

квадратное уравнение рисунок 93

И в этом случае квадратное уравнение будет иметь только один корень. Воспользуемся квадратным корнем:

квадратное уравнение рисунок 94

Далее выражаем x

квадратное уравнение рисунок 95

Это ещё одна формула для вывода корня квадратного корня. Рассмотрим её применение. Ранее мы решили уравнение x− 6+ 9 = 0, имеющее один корень 3. Решили мы его методом выделения полного квадрата. Теперь попробуем решить с помощью формул.

Найдём дискриминант квадратного уравнения. В этом уравнении = 1, = −6, = 9. Тогда по формуле дискриминанта имеем:

D = b2 4ac = (−6)− 4 × 1 × 9 = 36 − 36 = 0

Дискриминант равен нулю (= 0). Это означает, что уравнение имеет только один корень, и вычисляется он по формуле квадратное уравнение рисунок 96

квадратное уравнение рисунок 97

Значит корнем уравнения x− 6+ 9 = 0 является число 3.

Для квадратного уравнения, имеющего один корень также применимы формулы формула для вычисления первого корня квадратного уравнения и формула для вычисления второго корня квадратного уравнения. Но применение каждой из них будет давать один и тот же результат.

Применим эти две формулы для предыдущего уравнения. В обоих случаях получим один и тот же ответ 3

разложение квадратного трехчлена на множители рис 1

 

Если квадратное уравнение имеет только один корень, то желательно применять формулу квадратное уравнение рисунок 96, а не формулы формула для вычисления первого корня квадратного уравнения и формула для вычисления второго корня квадратного уравнения. Это позволяет сэкономить время и место.


Пример 3. Решить уравнение 5x− 6+ 1 = 0

Найдём дискриминант квадратного уравнения:

квадратное уравнение рисунок 98

Дискриминант больше нуля. Значит уравнение имеет два корня. Воспользуемся формулами корней квадратного уравнения:

квадратное уравнение рисунок 99

Значит корнями уравнения 5x− 6+ 1 = 0 являются числа 1 и одна пятая.

Ответ: 1; одна пятая.


Пример 4. Решить уравнение x+ 4+ 4 = 0

Найдём дискриминант квадратного уравнения:

квадратное уравнение рисунок 100

Дискриминант равен нулю. Значит уравнение имеет только один корень. Он вычисляется по формуле квадратное уравнение рисунок 96

квадратное уравнение рисунок 101

Значит корнем уравнения x+ 4+ 4 = 0 является число −2.

Ответ: −2.


Пример 5. Решить уравнение 3x+ 2+ 4 = 0

Найдём дискриминант квадратного уравнения:

квадратное уравнение рисунок 100

Дискриминант меньше нуля. Значит корней у данного уравнения нет.

Ответ: корней нет.


Пример 6. Решить уравнение (+ 4)= 3+ 40

Приведём данное уравнение к нормальному виду. В левой части располагается квадрата суммы двух выражений. Раскрóем его:

квадратное уравнение рисунок 101

Перенесём все члены из правой части в левую часть, изменив их знаки. В правой части останется ноль:

квадратное уравнение рисунок 102

Приведём подобные члены в левой части:

квадратное уравнение рисунок 103

В получившемся уравнении найдём дискриминант:

квадратное уравнение рисунок 104

Дискриминант больше нуля. Значит уравнение имеет два корня. Воспользуемся формулами корней квадратного уравнения:

квадратное уравнение рисунок 105

Значит корнями уравнения (+ 4)= 3+ 40 являются числа 3 и −8.

Ответ: 3; −8.


Пример 7. Решить уравнение квадратное уравнение рисунок 106

Умнóжим обе части данного уравнения на 2. Это позвóлит нам избавиться от дроби в левой части:

квадратное уравнение рисунок 107

В получившемся уравнении перенесём 22 из правой части в левую часть, изменив знак. В правой части останется 0

квадратное уравнение рисунок 108

Приведём подобные члены в левой части:

квадратное уравнение рисунок 109

В получившемся уравнении найдём дискриминант:

квадратное уравнение рисунок 110

Дискриминант больше нуля. Значит уравнение имеет два корня. Воспользуемся формулами корней квадратного уравнения:

квадратное уравнение рисунок 111

Значит корнями уравнения квадратное уравнение рисунок 106 являются числа 23 и −1.

Ответ: 23; −1.


Пример 8. Решить уравнение квадратное уравнение рисунок 112

Умнóжим обе части на наименьшее общее кратное знаменателей обеих дробей. Это позвóлит избавиться от дробей в обеих частях. Наименьшее общее кратное чисел 2 и 3 это число 6. Тогда получим:

квадратное уравнение рисунок 113

В получившемся уравнении раскроем скобки в обеих частях:

квадратное уравнение рисунок 114

Теперь перенесём все члены из правой части в левую часть, изменив у них знаки. В правой части останется 0

квадратное уравнение рисунок 115

Приведём подобные члены в левой части:

квадратное уравнение рисунок 116

В получившемся уравнении найдём дискриминант:

квадратное уравнение рисунок 118

Дискриминант больше нуля. Значит уравнение имеет два корня. Воспользуемся формулами корней квадратного уравнения:

квадратное уравнение рисунок 119

Значит корнями уравнения квадратное уравнение рисунок 112 являются числа две целых одна третьяи 2.


Примеры решения квадратных уравнений

Пример 1. Решить уравнение x= 81

Это простейшее квадратное уравнение, в котором надо определить число, квадрат которого равен 81. Таковыми являются числа 9 и −9. Воспользуемся квадратным корнем для их вывода:

квадратное уравнение рисунок 125

Ответ: 9, −9.


Пример 2. Решить уравнение x− 9 = 0

Это неполное квадратное уравнение. Для его решения нужно перенести член −9 в правую часть, изменив знак. Тогда получим:

квадратное уравнение рисунок 126

Ответ: 3, −3.


Пример 3. Решить уравнение x− 9= 0

Это неполное квадратное уравнение. Для его решения сначала нужно вынести x за скобки:

квадратное уравнение рисунок 127

Левая часть уравнения является произведением. Произведение равно нулю, если хотя один из сомножителей равен нулю.

Левая часть станет равна нулю, если отдельно x равно нулю, или если выражение − 9 равно нулю. Получится два уравнения, одно из которых уже решено:

квадратное уравнение рисунок 128

Ответ: 0, 9.


Пример 4. Решить уравнение x+ 4− 5 = 0

Это полное квадратное уравнение. Его можно решить методом выделения полного квадрата или с помощью формул корней квадратного уравнения.

Решим данное уравнение с помощью формул. Сначала найдём дискриминант:

D = b− 4ac = 4− 4 × 1 × (−5) = 16 + 20 = 36

Дискриминант больше нуля. Значит уравнение имеет два корня. Вычислим их:

квадратное уравнение рисунок 129

Ответ: 1, −5.


Пример 5. Решить уравнение квадратное уравнение рисунок 131

Умнóжим обе части на наименьшее общее кратное чисел 5, 3 и 6. Это позвóлит избавиться от дробей в обеих частях:

квадратное уравнение рисунок 132

В получившемся уравнении перенесём все члены из правой части в левую часть, изменив знак. В правой части останется ноль:

квадратное уравнение рисунок 133

Приведём подобные члены:

квадратное уравнение рисунок 134

Решим получившееся уравнение с помощью формул:

квадратное уравнение рисунок 135

Ответ: 5, минус пять шестых.


Пример 6. Решить уравнение x= 6

В данном примере как и в первом нужно воспользоваться квадратным корнем:

квадратное уравнение рисунок 137

Однако, квадратный корень из числа 6 не извлекается. Он извлекается только приближённо. Корень можно извлечь с определённой точностью. Извлечём его с точностью до сотых:

квадратное уравнение рисунок 139

Но чаще всего корень оставляют в виде радикала:

квадратное уравнение рисунок 138

Ответ: квадратное уравнение рисунок 140


Пример 7. Решить уравнение (2+ 3)+ (− 2)= 13

Раскроем скобки в левой части уравнения:

квадратное уравнение рисунок 142

В получившемся уравнении перенесём 13 из правой части в левую часть, изменив знак. Затем приведём подобные члены:

квадратное уравнение рисунок 143

Получили неполное квадратное уравнение. Решим его:

квадратное уравнение рисунок 144

Ответ: 0, −1,6.


Пример 8. Решить уравнение (5 + 7x)(4 − 3x) = 0

Данное уравнение можно решить двумя способами. Рассмотрим каждый из них.

Первый способ. Раскрыть скобки и получить нормальный вид квадратного уравнения.

Раскроем скобки:

квадратное уравнение рисунок 148

Приведём подобные члены:

квадратное уравнение рисунок 149

Перепишем получившееся уравнение так, чтобы член со старшим коэффициентом располагался первым, член со вторым коэффициентом — вторым, а свободный член располагался третьим:

квадратное уравнение рисунок 150

Чтобы старший член стал положительным, умнóжим обе части уравнения на −1. Тогда все члены уравнения поменяют свои знаки на противоположные:

квадратное уравнение рисунок 151

Решим получившееся уравнение с помощью формул корней квадратного уравнения:

квадратное уравнение рисунок 152

Второй способ. Найти значения x, при которых сомножители левой части уравнения равны нулю. Этот способ удобнее и намного короче.

Произведение равно нулю, если хотя бы один из сомножителей равен нулю. В данном случае равенство в уравнении (5 + 7x)(4 − 3x) = 0 будет достигаться, если выражение (5 + 7x) равно нулю, или же выражение (4 − 3x) равно нулю. Наша задача выяснить при каких x это происходит:

квадратное уравнение рисунок 154


Примеры решения задач

Предстáвим, что возникла необходимость построить небольшую комнату, площадь которой 8 м2. При этом длина комнаты должна быть в два раза больше её ширины. Как определить длину и ширину такой комнаты?

Сделаем примерный рисунок этой комнаты, который иллюстрирует вид сверху:

комната x на 2x рисунок 2

Обозначим ширину комнаты через x. А длину комнаты через 2x, потому что по условию задачи длина должна быть в два раза больше ширины. Множитель 2 и выполнит это требование:

комната x на 2x рисунок 1

Поверхность комнаты (её пол) является прямоугольником. Для вычисления площади прямоугольника, нужно длину данного прямоугольника умножить на его ширину. Сделаем это:

2x × x

По условию задачи площадь должна быть 8 м2. Значит выражение 2× x следует приравнять к 8

2x × x = 8

Получилось уравнение. Если решить его, то можно найти длину и ширину комнаты.

Первое что можно сделать это выполнить умножение в левой части уравнения:

2x2 = 8

В результате этого преобразования переменная x перешла во вторую степень. А мы говорили, что если переменная, входящая в уравнение, возведенá во вторую степень (в квадрат), то такое уравнение является уравнением второй степени или квадратным уравнением.

Для решения нашего квадратного уравнения воспользуемся изученными ранее тождественными преобразованиями. В данном случае можно разделить обе части на 2

квадратное уравнение рисунок 123

Теперь воспользуемся квадратным корнем. Если x= 4, то квадратное уравнение рисунок 40. Отсюда = 2 и = −2.

Через x была обозначена ширина комнаты. Ширина не должна быть отрицательной, поэтому в расчёт берём только значение 2. Такое часто бывает при решении задачи, в которых применяется квадратное уравнение. В ответе получаются два корня, но условию задачи удовлетворяет только один из них.

А длина была обозначена через 2x. Значение x теперь известно, подставим его в выражение 2x и вычислим длину:

2x = 2 × 2 = 4

Значит длина равна 4 м, а ширина 2 м. Это решение удовлетворяет условию задачи, поскольку площадь комнаты равна 8 м2

4 × 2 = 8 м2

Ответ: длина комнаты составляет 4 м, а ширина 2 м.


Пример 2. Огородный участок, имеющий форму прямоугольника, одна сторона которого на 10 м больше другой, требуется обнести изгородью. Определить длину изгороди, если известно, что площадь участка равна 1200 м2

Решение

Длина прямоугольника, как правило, больше его ширины. Пусть ширина участка x метров, а длина (+ 10) метров. Площадь участка составляет 1200 м2. Умножим длину участка на его ширину и приравняем к 1200, получим уравнение:

x(x + 10) = 1200

Решим данное уравнение. Для начала раскроем скобки в левой части:

квадратное уравнение рисунок 145

Перенесём 1200 из правой части в левую часть, изменив знак. В правой части останется 0

квадратное уравнение рисунок 146

Решим получившееся уравнение с помощью формул:

квадратное уравнение рисунок 147

Несмотря на то, что квадратное уравнение имеет два корня, в расчёт берём только значение 30. Потому что ширина не может выражаться отрицательным числом.

Итак, через x была обозначена ширина участка. Она равна тридцати метрам. А длина была обозначена через выражение + 10. Подставим в него найденное значение x и вычислим длину:

x + 10 = 30 + 10 = 40 м

Значит длина участка составляет сорок метров, а ширина тридцать метров. Эти значения удовлетворяют условию задачи, поскольку если перемножить длину и ширину (числа 40 и 30) получится 1200 м2

40 × 30 = 1200 м2

Теперь ответим на вопрос задачи. Какова длина изгороди? Чтобы её вычислить нужно найти периметр участка.

Периметр прямоугольника это сумма всех его сторон. Тогда:

P = 2(a + b) = 2 × (40 + 30) = 2 × 70 = 140 м.

Ответ: длина изгороди огородного участка составляет 140 м.


Задания для самостоятельного решения

Задание 1. Решить уравнение:
Решение:
Ответ: 2; −2.
Задание 2. Решить уравнение:
Решение:
Ответ: корней нет.
Задание 3. Решить уравнение:
Решение:
Ответ: 3; −3.
Задание 4. Решить уравнение, используя выделение полного квадрата:
Решение:
Ответ: 3; −13.
Задание 5. Решить уравнение, используя выделение полного квадрата:
Решение:
Ответ: 12; 4.
Задание 6. Решить уравнение, используя выделение полного квадрата:
Решение:
Ответ: 7; 5.
Задание 7. Решить уравнение:
Решение:
Ответ: 0; 1.
Задание 8. Решить уравнение:
Решение:
Ответ: 0; −3.
Задание 9. Решить уравнение:
Решение:
Ответ: 7; −7.
Задание 10. Решить уравнение:
Решение:
Ответ:
Задание 11. Решить уравнение:
Решение:
Ответ: 5; −5.
Задание 12. Решить уравнение:
Решение:
Ответ: 7; 2
Задание 13. Решить уравнение:
Решение:
Ответ: корней нет.
Задание 14. Решить уравнение:
Решение:
Ответ:
Задание 15. Решить уравнение:
Решение:
Ответ: 1; −5.
Задание 16. Решить уравнение:
Решение:
Ответ: 5; −9.
Задание 17. Решить уравнение:
Решение:
Ответ: −3; −4.
Задание 18. Решить уравнение:
Решение:
Ответ: .

Понравился урок?
Вступай в нашу новую группу Вконтакте и начни получать уведомления о новых уроках

Возникло желание поддержать проект?
Используй кнопку ниже